SlideShare a Scribd company logo
បជុំវ   សិស ពូែកគណិតវទ ក់ទី១០
                  បកែ បេ យ ែកវ សិរ"
បជុំវ             សិស ពូែកគណិតវទ                     ក់ទ១០
                                                                                                                         ី



                                  វ                 អូ           ពិក បៃពណីេវ ត
                                                                 ំ                                      មេលកទី XII                              ំ ២០០៦

                                              មុខ វ$ៈ គណិត វទ()ក់ទី ១០ រយៈេពល ១៨០/ទី
                                                    %

 .                             យ          ម               ( x + 1) x 2 − 2 x + 3 = x 2 + 1

    .        គ                                ABC            នប                ជង BC = a, CA = b, AB = c, B = 2 A, C = 4 A ,                                                   !

                                                                                                1   1   1 
             ង"ង#$ %ក '                        )(ន%ង R               គ     *            T = R2  2 + 2 + 2 
                                                                                               a   b   c 
+.           គ                                ABC            ម,                ង# A            ង"ង#$ %កក-.ង                                      ABC ប/ ជង AB

                  ង# T , ប* # CT
                           0                                 # ង"ង#        ង# K ផ2ងព T                           4ប           5 K 6ច!ន.ចក                            8 CT

             9)យ CT = 6 2                           ច: គ              * ប;<ងប                          ជង ប           #                         ABC

=.                    យប>ក#5 ច! @ គប# !A86ច!នBនគ # ប
                         ?                                                                                        # m,             នច!នBនគ # n C)មD

             n3 − 11n 2 − 87 n + m ;ចក                               ច# ន%ង 191

E.           គ         a , b, c > 0                       យប>ក# 5
                                                             ?
                      a4                                                            b4                                                           c4
                                                         +                                                        +                                                            ≤1
a   4
        +3   (a   6
                      +b   6
                               )( a   3
                                          +c    )
                                               3 2
                                                             b   4
                                                                     +3   (b   6
                                                                                   +c   6
                                                                                            )( b   3
                                                                                                       +a    )
                                                                                                            3 2
                                                                                                                          c   4
                                                                                                                                  +3   (c   6
                                                                                                                                                +a   6
                                                                                                                                                         )( c   3
                                                                                                                                                                    +b    )
                                                                                                                                                                         3 2


                                                                                   '()&


                                                                                        ចំេលយ

 .                             យ          ម              ( x + 1) x 2 − 2 x + 3 = x 2 + 1 (1)

             ;Cនក!               # ℝ

             Fង         x 2 − 2 x + 3 = t ច! @ t ≥ 2

             ព8 *                         ម         (1) G6 ( x + 1)t = x 2 + 1 (2)

             (2) ⇔ x 2 − 2 x + 3 − ( x + 1)t + 2( x − 1) = 0



2012-11-17បកែ បេ យៈ ែកវ សិរ                                                                                                                                                        Page 1
បជុំវ        សិស ពូែកគណិតវទ         ក់ទ១០
                                                                                ី


                                           t = 2
       ⇔ t 2 − ( x + 1)t + 2( x − 1) = 0 ⇔ 
                                           t = x − 1

      ច! @ t = 2 , យ)ង,ន                  x2 − 2 x + 3 = 2 ⇔ x2 − 2 x + 3 = 4

                           x = 1− 2
       ⇔ x2 − 2x − 1 = 0 ⇔ 
                           x = 1+ 2
                           
      ច! @ t = x − 1 , យ)ង,ន                     x2 − 2 x + 3 = x − 1

         x −1 ≥ 0                             x ≥ 1
       ⇔  2                                 ⇔       ⇔            x ∈∅
          x − 2 x + 3 = ( x − 1)              3 = 1
                                  2



      C:ច ន         !ន.!H     ប      #   ម           {
                                                  គI 1 − 2;1 + 2        }
                              a     b     c
 .     យ)ង     ន                 =     =      = 2R
                            sin A sin B sin C
            R   1    R   1    R   1
       ⇒      =     , =      , =
            a 2sin A b 2sin B c 2sin C
           1 1         1     1 
       ⇒ S=  2 +          +       
           4  sin A sin B sin 2 C 
                         2



              =
                  1
                  4
                    ( 3 + cot g 2 A + cot g 2 B + cot g 2C )
      ក-.ង ∆ABC         ន cot gA.cot gB + cot gB.cot gC + cot gC.cot gA = 1

                                             cot g 2α − 1
       9)យ យ)ង          ន cot g 2α =                      , ក-.ង * cot g 2α = 1 + 2cot gα .cot g 2α
                                              2cot gα
                  1
       ⇒ S=         3 + 3 + 2 ( cot gA.cot g 2 A + cot gB.cot g 2 B + cot gC.cot g 2C ) 
                  4                                                                     

                  1                                                             1
              =     6 + 2 ( cot gA.cot gB + cot gB.cot gC + cot gC.cot gA )  = (6 + 2) = 2
                                                                             4
                  4
+.     K 6ច! ន.ចក            8ប          # CT 9)យ L 6ច! ន.ចប/ ប                # ង"ង# Gន% ង ជង BC
             1
       * CK = CT               (*)       Jញ,ន L 6ច! ន.ចក                    8 BC ,
             2
                            1
              CL2 = CK .CT = CT 2 ,
                            2


2012-11-17បកែ បេ យៈ ែកវ សិរ                                                                           Page 2
បជុំវ       សិស ពូែកគណិតវទ           ក់ទ១០
                                                                                 ី


        I a 2 / 4 = 36 , I a = 12 (1)

      Lន.< នM ទ%        បទក:       .ន.    ក-.ង               BCT , យ)ង,ន

                CT 2 = BT 2 + BC 2 − 2 BT .BC.cos B

       ⇔ 72 = a 2 / 4 + a 2 − 144.cos B
                       3
       ⇔ cos B =         (2),       Fង (1)
                       2
      មO/ ង ទP , Lន.< នM ទ%              បទក:       .ន.   ក-.ង                ABC , យ)ង,ន

                b2 = c 2 + a 2 − 2ca.cos B ⇔ cos B = a / 2b (3)

      ព (1), (2), នQង (3) , យ)ង,ន                 (a, b, c) = (12,8,8)

=.    Fង P( x) = x 3 − 11x 2 − 87 x + m

       យ)ង,ន           P( x) ≡ ( x + a)3 + b (mod 191)

       ⇔ x3 + 3ax 2 + 3a 2 x + a3 + b ≡ x3 − 11x 2 − 87 x + m (mod 191)
         3a ≡ − 11 (mod 191) (1)
          2
       ⇔ 3a ≡ − 87(mod 191) (2)
         b ≡ m − a 3 (mod 191) (3)
         

       (1) ⇔ 3a ≡ 180 (mod 191) ⇔ a ≡ 60 (mod 191) ⇒ 3a 2 ≡ − 87(mod 191)

      C:ច ន        ∀m ∈ ℤ ,       នច!នBនគ # a, b C) មD              P( x) ≡ ( x + a)3 + b (mod 191)

           ងR    ម) 8 191 6ច! នBន;C8                នSង 191 = 3k + 2

                P(i) ≡ P( j ) (mod 191) ⇒ (i + a)3 ≡ ( j + a)3 (mod 191)

      Fង u = i + a , v = j + a , *

                u 3 ≡ v3 (mod 191) ⇒ u 3k ≡ v3k (mod 191)

                u 3k v 2 ≡ v3k + 2 ≡ v191 (mod 191) ≡ v (mod 191) ( ទ%           បទ Ferma )     (4)

       ⇒ v2 ≡ u 3k v3 ≡ u 3k + 3 (mod 191)

       ⇒ u 3k v2 ≡ u 3k .u 3k + 3 ≡ u 3k +1.u 3k + 2 ≡ u 3k +1.u191 ≡ u 3k +1.u (mod 191)


2012-11-17បកែ បេ យៈ ែកវ សិរ                                                                           Page 3
បជុំវ               សិស ពូែកគណិតវទ                             ក់ទ១០
                                                                                                                           ី


                                                   ≡ u 3k + 2 ≡ u191 ≡ u (mod 191) (5)

      ព (4) នQង (5) Jញ,ន u ≡ v (mod 191) ⇒ i ≡ j (mod 191)

      C:ច ន             ប) ∀i, j ∈ {1, 2, ... , 191} : i ≠ j (mod 191) *                                                P(i) ≠ P( j ) (mod 191)

      Jញ,ន                      ន n ∈ {1, 2, ..., 191} V/ ង                                         P(n) ≡ 191(mod 191) , I P (n) ⋮ 191

      C:ច ន             ច! @ គប# !A8គ # ប                                    # m, ; ង               នច!នBនគ # n C)មD                                       P (n) ⋮ 191

           (a       + b 6 )( a 3 + c3 ) =              (a         + b6 )( a 6 + 2a 3c3 + c 6 ) =                       (a       + b 6 )( a 6 + 2a 3c 3 + c 6 )
                                          2
E.     3        6                                  3         6                                                     3        6




                                =    3
                                         (a   12
                                                   + 2a3b6c3 ) + ( 2a 9c3 + b6c 6 ) + a 6b6 + a 6c 6

                                                                                                                       (a b         + a 2 c 2 ) = a 2b 2 + a 2 c 2
                                                                                                                                              3
                                ≥ 3 a 6b 6 + 3a 6b 4c 2 + 2a 6b 2c 4 + a 6c 6 =                                3         2 2



                                                             a4                                     a4                a2
      Jញ,ន                                                                            ≤                          = 2
                                a 4 + 3 ( a 6 + b 6 )( a 3 + c3 )
                                                                                  2        a 4 + a 2b 2 + a 2 c 2 a + b 2 + c 2

                                                             b4                                  b2
      C:ចW;C
          -                                                                           ≤
                                b 4 + 3 ( b6 + c 6 )( b3 + a 3 )
                                                                                  2        a 2 + b2 + c 2

                                                             c4                                c2
                                                                                      ≤
                                c 4 + 3 ( a 6 + b6 )( c3 + b3 )
                                                                                  2       a2 + b2 + c2

      ប:កLងXន%ងLងXAន                          មYពZង 8), យ)ង,ន

                                         a4                                                        b4                                                          c4
                                                                         +                                                            +                                                       ≤1 .
                      a +
                        4   3
                                (a   6
                                         +b   6
                                                  )( a   3
                                                             +c     )
                                                                   3 2
                                                                             b +
                                                                              4       3
                                                                                          (b   6
                                                                                                   +c   6
                                                                                                            )( b   3
                                                                                                                       +a   3 2
                                                                                                                                )         c +
                                                                                                                                          4       3
                                                                                                                                                      (a   6
                                                                                                                                                               +b   6
                                                                                                                                                                        )( c   3
                                                                                                                                                                                   +b    )
                                                                                                                                                                                        3 2



                                                                             '()&




2012-11-17បកែ បេ យៈ ែកវ សិរ                                                                                                                                                                   Page 4
បជុំវ             សិស ពូែកគណិតវទ                        ក់ទ១០
                                                                                                        ី



                       វ1       េសរ បឡងអូ                       ពិចេវ ត
                                                                ំ                              ម)ក់ទី១០ េលកទី XII

                                                                   វ              ទី១

                                             1
 .                យ      ម                     − x 1 − x2 = 1 − 2 x2                               (1)
                                             2
 .    ពQនQ    ប                      Q
                       ច!នBនពQ មQ នL<ជ?                   ន ; ប ប[8 a1 , a2 , ..., a9 មQនផ8ប:ក                         )( 1

                                                                (                 )
      Fង S K = ak + ak +1 + ak + 2 + ak +3 k = 1;6 , Fង M = max {S1 , S2 , S3 , S4 , S5 , S6 }

      ច: ក!       # !A8 : ចប! ផ.             ប       # M

+.     ក គប#Lន. គមនM f : ℝ → ℝ ផ0ង] # 8ក^
                                    0                                                 _

         (         )        (                        )
       f x3 − y + 2 y 3 f 2 ( x ) + y 2 = f ( y + f ( x ) ) ∀x, y ∈ ℝ (1)

                                                                           A    B 1
=.     គ                     ABC ផ0ង] # tan
                                      0                                      tan =                       យប>ក# 5 8ក^
                                                                                                            ?                 _ $!,ច#
                                                                           2    2 2
                                                                                                 A    B   C 1
      នQង គប# Wន# C) មD                                   ABC ;កងគI sin                            sin sin =
                                                                                                 2    2   2 10
                                                               '()&


                                                                           ចំេលយ

                                         (                     )
                                                                   2
              1              1
 .              − x 1 − x2 =                  1 − x2 − x
              2              2
      8ក^ខ        _ C) មD            ម           (1) ក!                #    1 − x2 ≥ 0 ⇔                 x ≤1   ព8 *
                                1
              (1) ⇔                  1 − x2 − x = 1 − 2 x2
                                 2
                                                                                          1
         ម          នH          ⇔ 1 − 2 x2 ≥ 0 ⇔                            x ≤
                                                                                           2

       យ)ង    ន        1 − 2 x2 ≥ 0 ⇒                1 − x 2 ≥ x C: ច *

              (2) ⇔             1 − x2 − x = 2             (    1 − x2 + x            )(       1 − x2 − x   )
2012-11-17បកែ បេ យៈ ែកវ សិរ                                                                                                        Page 5
បជុំវ       សិស ពូែកគណិតវទ              ក់ទ១០
                                                                                                ី


                   1 − x2 = x
                ⇔ 
                                (
                   2 1 − x2 + x = 1
                                                        )
                                             1
                   x ≥ 0            x = 2                  
                                                                   2
                                                            x =
                    1− x = x
                          2  2
                                      
                ⇔ 
                                                1                  2
                                    ⇔  x ≤                ⇔ 
                            1                  2                 2− 6
                   1 − x2 =     −x                         x =
                              2       2            1              4
                                        2 x − 2.x − 2 = 0
                                      

      C:ច ន             ម           (1)           នH         ព    x1 =
                                                                         2
                                                                          2
                                                                            ; x2 =
                                                                                   1
                                                                                   4
                                                                                       (   2− 6      )
 .              M = max {S1 , S 2 , S3 , S 4 , S5 , S 6 }
                                                                           9
       ⇒ 12 M ≥ 4S1 + S 2 + S3 + S 4 + S5 + S6 = 4∑ ai + a2 + 2a3 + 3a4 + 3a6 + 2a7 + a8 ≥ 4
                                                                          i =1

                  9
                                                                          1
      ( @        ∑ a = 1; a
                 i =1
                                    i   ≥ 0, i = 1;9 ) ⇒ M ≥
                                                                          3
                                                            1
                                             a1 = a5 = a9 =
           > " = " ក)
            a                           ន ព8                3
                                             a2 = a3 = a4 = a6 = a7 = a8 = 0
                                             
                                        1
      ច! 8)យ            M min =
                                        3
+.    ជ!នB      y = x3 ច: 8 (1) យ)ង,ន

                 f (0) + 2 x 3 ( 3 f 2 ( x) + x 6 ) = f ( x 3 + f ( x) )               (2)

      ជ!នB      y = − f ( x) ច:8 (1) , Jញ,ន

                 f ( x 3 + f ( x ) ) − 2 f ( x ) ( 3 f 2 ( x ) + f 2 ( x ) ) = f (0)           (3)

      ព (2) នQង (3) ⇒ 4 f 3 ( x) − 3 f 2 ( x).x3 − x9 = 0 ; ∀x

       ⇒     ( f ( x) − x )( 4 f
                            3               2
                                                ( x ) + x 3 f ( x ) + x 6 ) = 0 ; ∀x (4)

       ឃ)ញ5             4 f 2 ( x) + x3. f ( x) + x6 > 0 ; ∀x ≠ 0

      ព (4) យ)ង,ន                       f ( x) = x3 ; ∀x

2012-11-17បកែ បេ យៈ ែកវ សិរ                                                                              Page 6
បជុំវ          សិស ពូែកគណិតវទ          ក់ទ១០
                                                                                        ី


             កជ!នB                 Q
                           ច:8 c)ង<ញ, ផ0ង] #8ក^ខ
                                           0                                _ បdន

      ច! 8)យ               f ( x) = x 3
                                                    A                B
=.     យ)ង         ន        S = p ( p − a ) tan       = p( p − b) tan =           p ( p − a )( p − b)( p − c)
                                                    2                2
            A   B        S2            p−c a+b−c
       ⇒ tan tan = 2                 =    =
            2   2 p ( p − a)( p − b)    p   a+b+c
             A    B 1
       tan     tan =               ⇔ 2(a + b) − 2c = a + b + c ⇔ a + b = 3c (1)
             2    2 2
                                           abc                        r
      មO/ ង ទP               S = pr =               ⇒       p.abc.      = S 2 = p( p − a )( p − b)( p − c) (2)
                                           4R                        4R
              p = 2c                                                     r
       (1) ⇔                               ព (2) Jញ,ន ab                   = p 2 − ( a + b ) p + ab
             p −c = c                                                   4R
             r 
       ⇒ 1 −    ab = 2c
                          2
                                              (3)
          4R 
                                  A    B   C 1                         r   1
      Fមប! Sប#              sin     sin sin =   ⇒                        =
                                  2    2   2 10                       4 R 10
                                             20 2
      ព (3) យ)ង,ន                    ab =      c , មWន% ង (1) យ)ងJញ,ន a នQ ង b 6ប
                                                   B -                                                           H   ប     #
                                             9
                                          20 2         5       4
         ម             t 2 − 3ct +          c = 0; t1 = c; t2 = c
                                          9            3       3
                                                  5
                                              a = 3 c
                                              
      យក a ≥ b , យ)ង,ន                        
                                              b = 4 c
                                              
                                                  3
                                      2                 2
                 4          5 
       ⇒ b + c =  c  + c2 =  c  = a2
               2       2
                                                                      ⇒ ∆ABC ;កង           ង# A
                 3          3 
      ផ0.យមក< Qញ, 4ប                  5 ∆ABC ;កង                ង# A        យ)ង,ន

                   
                   a = 2 R            a = 2 R                                        a = 2 R
                    2                                                                
                   a = b + c    (1) ⇒ (3c − b) 2 = b 2 + c 2                       ⇒ b = 4 r
                           2  2

                       1              b = 4r ( p = 2c)                               c = 3r
                    S = bc = pr                                                      
                       2

2012-11-17បកែ បេ យៈ ែកវ សិរ                                                                                              Page 7
បជុំវ            សិស ពូែកគណិតវទ              ក់ទ១០
                                                                                              ី


                                   a = 2 R                                          r 2       A   B   C 2
      ព (1) យ)ង,ន                          ⇒ 5r = 2 R ⇒                              =  ⇒ 4sin sin sin =
                                   a = 5r                                           R 5       2   2   2 5
                   A    B   C 1
       ⇒ sin         sin sin =
                   2    2   2 10
      (ប> ប
         e              #8!f #           g<,ន              យប>ក#)
                                                              ?
                                                                   '()&


                        វ1         េសរ បឡងអូ                       ពិចេវ ត
                                                                   ំ                   ម)ក់ទី១០ េលកទី XII

                                                                   វ             ទី២

                                                                   x 2004 + 16
 .     គ     <Q     ម            a + 1 − x 2006 +                                      ≤ 2 x 2004 + 16
                                                               a + 1 − x 2006
       កច! នBនពQ            a ធ! ប!ផ.    C)មD         <Q       ម             នH      ; ព ប/.       i

 .     គ          m, n 6ព ច!នBនគ #< Qជ?                    ន                     យ     ម

                                        1                                    1
                  sin 2 n x +                     = cos 2n +
                                 sin 2 m +1 x                          cos 2m +1 x
+.     គ          I 6ផjQ       ង"ង#$ កក-.ង
                                     %                                 មQន       ម,      ABC           M 6ច!ន.ចក          8 IC ,

       N 6ច!ន.ចក                 8ប           # AB             J 6ច!ន.ចក               8ប         # MN     Fង x, y , z 6ប* #
                                                                                                                          0

      ប PងW-                #Fម A, B, C 9)យប* # នមBយk;ចកប Q
                                             0                                                                    ABC 6ព ;ផ-ក

           )W
            ( -             យប>ក# 5 4 ប* # x, y , z នQង IJ
                               ?        0                                                       #Fមច!ន.ច BមWមBយ
                                                                                                            -

=.     គ                      Q
                  n 6ច!នBនគ #<ជ?              ន       ពQ នQ        FSង               2n ជB CកនQង 2n ជB ឈ                បm

      នមB យk            ន               ច! នBនមB យច:8 ;C8ច! នBន *                          Qn    oក-.ង        {                    }
                                                                                                           !ន.! 1, 2, 3, ..., 4 n 2 ,

       ប m ព ផ2ងW-                                ច!នBន ផ2ងW-                    កច!នBន N ធ!ប!ផ. ;C8              ន8ក^         p

      ច! @ គប# បPប                            ច! នBនC: ចZង 8)                     នជB CកមB យ ជB ឈ មB យ ;C8 o 8)
                                                                                             I

      ជB Cក IជB ឈ *                         នព ច! នBន p, q ផ0ង] # | p − q | ≥ N
                                                                0
                                                                   '()&

2012-11-17បកែ បេ យៈ ែកវ សិរ                                                                                                            Page 8
បជុំវ            សិស ពូែកគណិតវទ                         ក់ទ១០
                                                                                                            ី



                                                                         ចំេលយ

                                   (                                                   )
                                                                                           2
                                       a + 1 − x 2006 − x 2004 + 16
 .                c)ង< Qញ                                                                      ≤ 0 ច! @ x 2 ≤ 1
                                                   a + 1− x          2006



       ប) a < 0 < Q         ម          a + 1 − x 2006 < 0                   នច! 8)យSប# មQនL                          #

       ប) a ≥ 0 យ)ង,ន                     ម              a = 16 + x 2004 − 1 − x 2006

      Lន. គមនM oLងXZង                            ! 6Lន. គមនM គ:                Lន. គមនM ក) ន 8) [0;1] C:ច *                               ម     នH

       ព8 a           Qn     oច * 3, 17  , Jញ,ន !A8ធ! ប!ផ.
                                 q                                                                           ប         # a គI   17 ,

       ព8 * H                   J!ងព គI x = −1, x = 1

 .    Fង u = sin x, v = cos x; u.v ≠ 0 9)យ u 2 + v 2 = 1
                                                       1                      1
       យ)ង,ន                ម           u 2n +         2 m +1
                                                                = v2n +       2 m +1
                                                   u                      v
       ប) uv < 0 LងX ^ ង : ច6ង 1 , LងX ^ ង ទP ធ!6ង 1 , មQ ន ផ0ង] #
                                                                 0
                                        1
      Lន. គមនM y = x 2 n +              2 m +1
                                                 ច. o 8) [−1;0)                        C:ច ន             ប) u, v < 0 ; u ≠ v គIមQន ផ0ង] #
                                                                                                                                        0
                                   x
                                                 យ)ង,ន ( u 2 n − v 2 n ) ( u.v )
                                                                                                2 m +1
      ពQនQ      u, v > 0; u ≠ v                                                                          = u 2 m +1 − v 2 m +1

         (                                                                                          )
       I u 2 n −1 + v 2 n −1 + uv ( u 2 n − 3 + v 2 n − 3 ) + ... + (uv) n −1 (u + v) (uv) 2 m +1 = u 2 m + v 2 m + u 2 m −1v + ...
                                             n−2      n−2     2 m +1         2 m +1
                      1  1 2         1       1   1            1          1
                                                                                         2m

      LងXZង ឆ"ង :ច6ង 1 + +   + ... +   +                   < 2          = 
                      2  2
                                        2      2   2 
                                                                        2         2
                                                                  m −1
                                                 1
      LងXZង                !ធ!6ង u 2 m + v 2 m ≥                            C: ច * LងXZង                          ! : ច6ងLងXZង ឆ"ង
                                                 2
                                                       π
      C:ច ន        sin x = cos x I x =                   + kπ
                                                       4
+.    Lemma:                                ABC , Fង A ', B ', C ' PងW6ច!ន.ចក
                                                                      -                                                     8ប       #ប       ជង BC ,

       CA, AB       ប            ប* #
                                   0                #Fមក!ព:8នមB យ ប                             #                   A ' B ' C ' 9)យ;ចកប Q

                      A ' B ' C ' 6ព              ប        ព"W-          ង#ច!ន.ច;C86ផjQ                       ង"ង#$ %កក-.ង                    ABC


2012-11-17បកែ បេ យៈ ែកវ សិរ                                                                                                                      Page 9
បជុំវ         សិស ពូែកគណិតវទ               ក់ទ១០
                                                                                            ី


      ពQ 6C:ច ន : Fង M 6ច!ន.ច                               Qn       o 8) B ' C ' V/ ង             A ' M ;ចកប Q

       A ' B ' C ' 6ព       * MC ' = p '− b ' 9)យ MB ' = p '− c ' ;C8 p ' = (a '+ b '+ c ') / 2

                      ( p '− c ') A′C ′ + ( p′ − b′) A′B′ ( p − c) A′C ′ + ( p − b) A′B′
      Jញ,ន                  A′M =                        =                               =
                                        a'                               a
                                                                                         A
                        ( p − c) AC + ( p − c ) AB
                     =−
                                      2a
      មO/ ង ទP , យ)ង ន                                                            C' M                                      B'
                                                                                                                    I
                            2 p A′I = a A′A + b A′B + c A′C
                                                                                                                                   C
                                                (                )
                                                                                                         B          A'
                                              a AB + AC + (b − c) BC
                                         =−                                        = −( p − b) AB − ( p − c) AC
                                                                 2
      C:ច ន     A ', I , M       #       ង# ជB W-

         cប#មក8!f #< Qញ                    Fង G 6ទ បជ.! ទ!នង#                                ABC             យ J 6ទ បជ.!ទ!ងន#

       ប     #បBនច! ន.ច I , A, B, C * I , G, J ង#                       ង# ជB W-

      Fមចxប# ប!;8ង$!ងផjQ                       G Fមផ8 ធPប −1 / 2 ប!;8ង A G6 A ', B G6 B ', C G

      6 C'      ប!;8ងប* # J!ងប x, y , z G6ប* #J!ងប (Fម Lemma) ប
                       0                    0                                                                     ព"W-      ង# I

      ប:ក BមJ!ង I , G, J             #    ង# ជB W Jញ,ន x, y, z, IJ
                                                 -                                        # Fមច!ន.ច មWមB យ
                                                                                                    B -

=.    ពQនQ     បPប PបC:ចZង                            ម

                2n 2 − n + 1                  ...     ...        2n 2              ...                            4n 2

                        .

                        .

                        .

                   2n + 1

                    n +1                                         2n         2n 2 + n + 1           ...          2n 2 + 2n

                        1                     2       ...         n           2n 2 + 1             ...          2n2 + n


2012-11-17បកែ បេ យៈ ែកវ សិរ                                                                                                  Page 10
បជុំវ      សិស ពូែកគណិតវទ              ក់ទ១០
                                                                               ី


       ប)     ន i, j o 8)ជB CកV/ ង                     i − j ≥ N * 2n 2 + n − 1 ≥ N

       ប) o 8)ជB ឈ           * 2n 2 − n ≥ i − j ≥ N           C:ច ន         N ≤ 2n 2 + n − 1

      ពQនQ     N = 2n 2 + n − 1

      Fង A = {1, 2, ..., n 2 − n + 1} ; B = {3n 2 , 3n 2 + 2, ..., 4n 2 }     ច! @ គប# i       Qn   oក-.ង A, j     Qn

       oក-.ង B , យ)ង,ន i − j ≥ 3n 2 − ( n 2 − n + 1) = 2n 2 + n − 1                   (**)

      ច! @ ជB Cក ;C8              នផ0.កd . ប         # A , យ)ង y56ជB C ប ភទទ , ជB ឈ ;C8

            នផ0.កd . ប      # A    g<,ន y56ជB ឈ ប ភទទ                               ច! @ ជB Cក ;C8        នផ0.កធ

      d . ប       # B , យ)ង y56ជB Cក ប ភទទ , ច! @ ជB ឈ ;C8                                     នផ0.កd . ប        # B

       យ)ង y56ជB ឈ ប ភទទ

      Fង p, q PងW 6ច!នBនជB Cក នQ ងជB ឈ ប ភទទ
                 -                                                            *      p.q ≥ n2 − n + 1 , C:ច *

                p + q ≥ 2 pq ≥ 4n 2 − 4n + 4 > 2n − 1

      Fង r , s PងW 6ច!នBនជB C , នQ ងជB ឈ ប ភទទ
                  -                                                           * r.s ≥ n 2 + 1 , C:ច *

               r + s ≥ 2 rs ≥ 4n 2 + 4 > 2n
      C:ច ន      p + q + r + s ≥ 4n + 1 , Jញ,ន              នជB Cក ជB ឈ ;C86 ប ភទទ ផង នQ ង6
                                                                   I

       ប ភទទ ផង            ព (**) Jញ,ន !A8ធ!ប!ផ.                 ប     # N គIp 2n 2 + n − 1
                                                  '()&




2012-11-17បកែ បេ យៈ ែកវ សិរ                                                                                 Page 11
បជុំវ      សិស ពូែកគណិតវទ         ក់ទ១០
                                                                         ី



                    វ1      េសរ បឡងអូ           ពិចេវ ត
                                                ំ                ម)ក់ទី១០ េលកទី XII

                                                វ         ទី៣

 .     ក គប# !A8 a, b, c, d , e ∈ [0,1] C)មD

                a        b        c        d        e
       A=           +        +        +        +         =4
            1 + bcde 1 + cdea 1 + deab 1 + eabc 1 + abcd
 .     គ      f ( x) = ax3 + bx 2 + cx + d V/ ង                 f ( x) ≤ 1, ∀x ∈ [ −1,1] (1)

       កច! នBន ថ k : ចប! ផ.        C) មD     3ax 2 + 2bx + c ≤ k , ∀x ∈ [ −1,1], ∀f ផ0ង] # (1)
                                                                                         0

+.    ក-.ងបqង#, គ                    ម| ង2 ABC ផjQ      O     ប* # ( d ) < Q8ជ.!<ញ O
                                                                0                Q              #ប         ប* #
                                                                                                             0

                                                                               1          1          1
       BC , CA, AB PងW-         ង# M , N , P          យប>ក# 5 T =
                                                         ?                           +          +
                                                                              OM 4       ON 4       OP 4
      គIមQន; ប ប[8

=.          យប>ក#5 ច! @ គប# !A8គ # ប
               ?                                         # m,      នច! នBនគ # n C)មD

       n3 − 11n 2 − 87 n + m ;ចក       ច# ន%ង 191
                                               '()&


                                                    ចំេលយ

 .         យមQន ធ")     , #បង# 8ក^          ទ: G 4ប         5 a ≤ b ≤ c ≤ d ≤ e (*)
                              a         b         c         d         e      a+b+c+d +e
       ព8 *         A≤            +         +         +         +          =
                         1 + abcde 1 + abcde 1 + abcde 1 + abcde 1 + abcde     1 + abcde
           យ a, b, c, d , e ∈ [ 0,1] * :

               (1 − abc)(1 − de) + (1 − ab)(1 − c) + (1 − d )(1 − e) + (1 − b)(1 − a) ≥ 0 (1)

           ⇒ a + b + c + d + e ≤ 4 + abcde ≤ 4(1 + abcde)             (2) .

      C:ច *      A≤4        ប) A = 4 *         មYព ក)            ន o (1) នQង (2) Jញ,ន




2012-11-17បកែ បេ យៈ ែកវ សិរ                                                                                 Page 12
បជុំវ             សិស ពូែកគណិតវទ             ក់ទ១០
                                                                                      ី


              abcde = 0 ⇒ a = 0
              de = 1
              
                                  a = 0
              c = 1             ⇒ 
              d = 1 ∨ e = 1       b = c = d = e = 1
              
              b = 1
              
            0      Q
       ផ0ង] # c)ង<ញ              ព8 a = 0, b = c = d = e = 1 *                     A=4

      C:ច ន ប              បព|ន} !A8;C8                 g< ក (a, b, c, d , e) 6ប          ច!~     #ប   # (0,1,1,1,1)

 .    Fង A = f (−1), B = f (−1 / 2), C = f (1 / 2), D = f (1) *
                   2   4   4   2                                     2   2   2   2
              a = − A + B − C + D,                              b=     A− B − C + D
                   3   3   3   3                                     3   3   3   3
                         A 4  4  D                              A 2  2   D
              c=          − B+ C− ,                     d =−     + B+ C−
                         6 3  3  6                              6 3  3   6
                                                    A                      4B
              h( x) = 3ax 2 + 2bx + c = −             (12 x 2 − 8 x − 1) +    (3 x 2 − x − 1) −
                                                    6                       3
                                                                     4C                  D
                                                                        (3 x 2 + x − 1) + (12 x 2 + 8 x − 1)
                                                                      3                  6
      Fមប! Sប# Jញ,ន                A , B , C , D ≤ 1 , C:ច * , ប) ∀x ∈ [ −1,1] គ,ន
                           1                   4               4              1
               h( x ) ≤      12 x 2 − 8 x − 1 + 3 x 2 − x − 1 + 3x 2 + x − 1 + 12 x 2 + 8 x − 1
                           6                   3               3              6
           យ A + B = max ( A − B , A + B ) * ច! @ ∀x ∈ [ −1,1] យ)ង,ន

                                                                 (
              12 x 2 + 8 x − 1 + 12 x 2 − 8 x − 1 = max 16 x , 24 x 2 − 2 ≤ 22        )
                                                            (
               3 x 2 + x − 1 + 3 x 2 − x − 1 = max 2 x , 6 x 2 − 2 ≤ 4         )
                         22 16
       ⇒    h( x ) ≤       + = 9, ∀x ∈ [ −1,1]
                         6   3
      ច! @     f ( x) = 4 x3 − 3x * ∀x ∈ [ −1,1] Fង x = cos t យ)ង,ន f ( x ) = cos 3t ≤ 1

       9)យ max 3ax 2 + 2bx + c = max 12 x 2 − 3 = 9
               [ −1,1]                        [ −1,1]

      Jញ,ន ច! នBន ថ k : ចប!ផ. ;C8 ផ0ង] #
                                       0                                   !       ) 8!f # គI 9




2012-11-17បកែ បេ យៈ ែកវ សិរ                                                                                            Page 13
បជុំវ      សិស ពូែកគណិតវទ         ក់ទ១០
                                                                           ី


+.    Fង A ', B ', C ' PងW6ច!ន.ចក
                          -                       8 BC , CA, AB * OA ' = OB ' = OC ' = R / 2
                                                                          2π
              Q
      Fង i 6<.ចទ| •កF ប                 # (d )         (       )
                                                 Fង i , OA′ = α , x =
                                                                           3
                                                                                  *                    A

               ( i , OB′) = ( i , OA′) + (OA′, OB′) = α + x + k 2π
               ( i , OC′) = ( i , OA′) + (OA′, OC′) = α − x + k 2π                              C'
                                                                                                       O
                                                                                                            B'
                                                                                                             N
                          OA ' 4  OB ' 4  OC '  4                                   P
                       1
               T=      2        +       +       
                   R   OM   ON   OP  
                                                                    M                                          C
                                                                                     B              A'
                  2
                     16
                 =       cos 4 α + cos 4 (α + x) + cos 4 (α − x) 
                       4                                         
                     R
                     4
                 =       (1 + cos 2α ) 2 + (1 + cos(2α + 2 x)) 2 + (1 + cos(2α − 2 x)) 2 
                       4 
                                                                                          
                     R
                     4
                 =      [3 + 2(cos 2α + cos(2α + 2 x) + cos(2α − 2 x))] +
                     R4
                            4
                        +       cos 2 2α + cos 2 (2α + 2 x) + cos(2α − 2 x)) 
                              4                                                     (1)
                            R
       2sin x [ cos 2α + cos(2α + 2 x) + cos(2α − 2 x ) ] =

               = [sin(2α + x ) − sin(2α − x) + sin(2α + 3 x ) − sin(2α + x) + sin(2α − x) − sin( 2α − 3 x ) ]

               = sin(2α + 3x) − sin(2α − 3x) = sin(2α + 2π ) − sin(2α − 2π ) = 0 (2)
                                                      1
       cos 2 2α + cos 2 (2α + 2 x) + cos 2 (2α − 2 x) =  [3 + cos 4α + cos(4α + 4 x) + cos(4α − 4 x)]
                                                       2
       2sin 2 x [ cos 4α + cos(4α + 4 x ) + cos(4α − 4 x )] = [sin(4α + 4π ) − sin(4α − 4π )] = 0 (3)

                                    4      3  18
      Fម (1),(2),(3) ⇒ T =           4 
                                         3 +  = 4 មQន; ប ប[8
                                    R      2 R

=.    (8!f # ន             g<,ន ជ)      6<Q>a         បcង- ម)8ច! 8)យ o<Q>a                           បcង)
                                                  '()&




2012-11-17បកែ បេ យៈ ែកវ សិរ                                                                                 Page 14
បជុំវ        សិស ពូែកគណិតវទ             ក់ទ១០
                                                                                       ី



                       វ1          េសរ បឡងអូ              ពិចេវ ត
                                                          ំ                   ម)ក់ទី១០ េលកទី XII

                                                           វ            ទី៤

 .    Fង ជ)ងប               ក! ព    #ក.-ង                  ABC           យ A ', B′, C ′           គ        *ប       ម.! ប       #

                    A ' B ' C ' 6Lន.គមនMAនប                         ម.! A, B, C              យប>ក# 5 ម.!ធ!ប!ផ.
                                                                                                ?                           ប         #

                    A ' B ' C ' V/ ង fច                        ក•      )(ន%ងម.! ធ!ប!ផ.       ប    #              ABC

          ) ព8            ;C8 ក)        ន           មYព?

                                                         2 9 xyz
 .             យប>ក#5
                  ?                xy + yz + zx ≤          +     , ក-.ង * x, y , z 6ប                       ច!នBនពQ មQន
                                                         7   7
      L< Qជ?    ន ផ0ង] #8ក^
                       0                     _      x + y + z =1
                                                                                             1 1 1
+.     គ        a, b, c 6ប          ច!នBនគ # < Qជ?        ន ផ0ង] # 8ក^
                                                                 0                       _    − = 9)យ d 6 B;ចក
                                                                                             a b c
       មធ! ប!ផ.
       B            ប       #ពBកƒ                យប>ក#5 abcd នQង d (b − a ) 6ប
                                                    ?                                                       ច!នBន        ,កC

                                                          xy + yz + zx = 12
=.             យប>ក#5 បព| ន}
                  ?                          ម                                              នច! 8)យ; មB យគ # ក.-ង                   !ន.!
                                                          xyz − x − y − z = 2
      ប                  Q
                ច!នBនពQ <ជ?         ន               យប>ក#5 បព|ន}
                                                       ?                          នច! 8)យច! @              x, y , z 6ច!នBនពQ

       ផ2ងW-

                                                         '()&


                                                               ចំេលយ

 .    ពប         ច .            $ កក-.ង, យ)ង,ន ប
                                  %                                       ម.! ប/.នWC: ច បZង
                                                                                   -    :                   ម
                                                                                                             A
       yប           ម.!      យ α , β , γ C:ច :ប
                                                                                                                    B'
      C:ច *        យ)ង,ន A ' = 2α , B ' = 2β , C ' = 2γ
                                                                                                      C'
       9)យ A = β + γ , B = γ + α , C = α + β

      Jញ,ន                A ' = B + C − A; B = C + A − B, C = A + B − C
                                                                                              B             A'                              C

2012-11-17បកែ បេ យៈ ែកវ សិរ                                                                                                         Page 15
បជុំវ          សិស ពូែកគណិតវទ          ក់ទ១០
                                                                                        ី


      „c:< យ)ង          ងR      ឃ)ញ5                  A' ≥ B ' ⇔ B + C − A ≥ C + A − B ⇔ B ≥ A

      C:ច * , ប) A ≥ B ≥ C * C ' ≥ B ' ≥ A '

      *!        A ≤ C' ⇔         A ≤ A + B − C ⇔ B ≥ C : ពQ

           >a    មYព ក)              ន ព8                           ABC 6                     ម|ង2
                7        9x           9 xyz
 .     ប) x ≥     , * 1≤    Jញ,ន xy ≤
                9        7              7
                                                 2                         2 2
      6ង ន G ទP                ( y + z) ≤          , *         xy + xz <    <
                                                 9                         9 7
                                                                                      2 9 xyz
      C:ច * ក-.ងក                ន            យ)ងទទB8,ន xy + yz + zx <                  +
                                                                                      7   7
                                7                9x
      „c:<4ប        5 x<          , 9)យC:ច * 1 −    >0
                                9                7
                             (1 − x) 2
       យ)ង,ន yz ≤
                                 4
                                                                                9 x  (1 − x)
                                                                                               2
                                                                                                              2
      ព*         យ)ង Wន#;                g<              យប>ក#5
                                                            ?                   1−             + x(1 − x) ≤
                                                                                  7  4                      7

                ⇔ (7 − 9 x)(1 − x)2 + 28(1 − x) ≤ 8 .

                ⇔ 9 x3 + 3x 2 − 5 x + 1 ≥ 0 ⇔ ( x + 1)(3x − 1)2 ≥ 0 .
                                                                                                                        1
      <Q   មYព ន ពQ 6នQចj ( @                                   Q
                                                        x មQ នL<ជ?        ន), ច! @         មYព ក)         ន ព8 x =
                                                                                                                        3
      C:ច ន < Q     មYព           g<,ន                 យប>ក#, ច! @
                                                          ?                       >a       មYព ក)        ន8. F;
                    1
       x= y=z=
                    3
                  a      b     c
+.    Fង A =        , B = , C = , C:ច *                         A, B, C        ន B;ចក Bមធ!ប!ផ. គI 1
                  d      d     d
                    1 1 1
       យ)ង,ន         − = , *                          AB = C ( B − A)
                    A B C
       យ)ង        យប>ក#5 ( B − A)
                     ?                                       g<; 6ច! នBន          ,កC        ប) ផ.0យមក< Qញ, គI     g<   នច! នBន

      ប…ម p មB យV/ ង                                  |យគ.
                                                       "        ធ!ប!ផ.     ប     # p ;ចក     ច# ន%ង ( B − A) គI p 2 r +1 ច! @ r


2012-11-17បកែ បេ យៈ ែកវ សិរ                                                                                             Page 16
បជុំវ           សិស ពូែកគណិតវទ           ក់ទ១០
                                                                                    ី


      6ច! នBន         មBយ

      „c:< ប) p r +1 ;ចក           ច# A , * ƒក•;ចក                 ច#     B = ( B − A) + A

      Jញ,ន p 2 r + 2 ;ចក              ច# AB         ;     p មQន†ច;ចក            ច# C ( @          A, B, C មQន    នកF ម),
                                                                                                                     B

      Jញ,ន p 2 r + 2 ;ចក              ច# ( B − A)          មQន     ម 9 .ផ8! C:ច * ;C8†ច G ច * គI
                                                                                          B

       p r ;ចក       ច# A , 9)យ យ)ងក•,ន8ទ}ផ8C: ចWច! @
                                                 -                                  B      ព * ;C8†ច G ចគI
                                                                                                       B

       p 2r ;ចក      ច# ន%ង AB , ចគI;ចក
                                 B                      ច# ( B − A)      មQ ន   ម 9 .ផ8! ក               * បˆញ5
                                                                                                             e

       ( B − A) 6ច! នBន            ,កC          ;       ABC ( B − A) = ABAB, *             ABC ( B − A)       g<; 6ច! នBន
                                  ABC ( B − A)
                ,កC, BចគI                      = ABC ក•6ច!នBន                     ,កC;C
                                    B−A
      C:ច * (b − a)d = d 2 ( B − A) នQ ង abcd = d 4 ABC                         .ទ}; 6ច!នBន          ,កC

=.    6ក#;        ង (2,2, 2) 6ច! 8)យមB យ (ក-.ង                   !ន.!ប     ច! នBនពQ <Qជ?     ន)

          ប) យ)ង$ # ទ.ក5 z C:ច6,នC% ង !A8, * គI យ)ង,ន

                                    11z − 2
                           x + y = z2 + 1
                           
                           
                            xy = z + 2 z + 12
                                   2

                           
                                     z2 + 1
      C:ច * 8ក^ខ                 _ $!,ច# នQ ង គប# Wន# C)មD                  x, y , z 6ប      ច! នBនពQ < Qជ?      នគI

                           (11z − 2)2 > 4( z 2 + 2 z + 12)( z 2 + 1)             (*) .
                                                                                   2
      (ƒd*,ន5 x នQង y                     .ទ}; 6ច! នBនពQ ) 9)យ z >                   ( C) មD        x, y នQង z           Q
                                                                                                                   .ទ}; <ជ?
                                                                                  11
           ន)     ព8 * (*) q យ G6

                           4 z 4 + 8 z 3 − 69 z 2 + 52 z + 44 ≤ 0 ⇔ ( z − 2)2 (2 z + 11)(2 z + 1) ≤ 0
                                                          11        1
      ក          ន    g<,ន ផ0ង] # ព8 −
                                0                            ≤ z ≤ − នQង z = 2
                                                           2        2
          *      នH       <Qជ?    ន; មBយគI z = 2 ( ចព *
                                                   B                      x = 2, y = 2 , 6ង ន G ទP                     យ

      ;    បព|ន}      ម          ន ឆq. ច! @         x, y នQង z )


2012-11-17បកែ បេ យៈ ែកវ សិរ                                                                                            Page 17
បជុំវ         សិស ពូែកគណិតវទ          ក់ទ១០
                                                                               ី


      ច! @ ប            H    6ច!នBនពQ           ផ2ង ទP        យ)ងគB ពQនQ     ម) 8     ង# ប         ! A8   Qn     oច *q
            11     1
      ព −      G −
             2     2
                                                                             11
      ច! @       z = −1 យ)ង,ន,នH                  មQ ន ផ2ងW (−1, −1, −
                                                           -                    ) , $8
                                                                              2
                                                12 + 2 21      12 − 2 21
      ច! @       z = −2 * ,ន x = −                        ,y=−           , ពBកƒ            .ទ};   ផ2ងW-
                                                     5              5
                                                     '()&


                       វ1      េសរ បឡងអូ             ពិចេវ ត
                                                     ំ                   ម)ក់ទី១០ េលកទី XII

                                                         វ         ទី៥

                                                              1 1 1
 .     គ             Q
             3 ច!នBន<ជ?        ន a, b, c ផ0ង] #
                                              0                + + =3                យប>ក#5
                                                                                        ?
                                                              a b c
                 4       4       4          3        3         3
                     a 3 + b3 + c 3 ≥ a 2 + b 2 + c 2
                                               x1 + x2 − x3 x4 < 0
                                              
                                              ( x1 + x2 )( x3 + x4 ) − x1 x2 − x3 x4 < 0
 .                   យ បព|ន}     ម            
                                              ( x1 + x2 ) x3 x4 − ( x3 + x4 ) x1 x2 < 0
                                               x1 > 0, x2 > 0, x3 > 0, x4 > 0
                                              
+.     ក គប#ប           ច! នBនគ # ធម(6 Q;C8                  ន 8ខបខ0ង# C)មD         ច!នBននមB យk6មធ មនព"ន

       ប    #ប         ច!នBន;C8Jញ ចញពច!នBន *                               Q
                                                                         យ<ធច! ~      #ប          8ខ ប         #ច!នBន

       *

=.    គ      *ច! នBន ជង ប            #ព9.            នQយ| ;C8            នក!ព:8 4 W A, B, C , D ផ0ង] #
                                                                                   -                 0

                             1   1   1
      ទ!*ក# ទ!នង               =   +
                            AB AC AD
                                                     '()&




2012-11-17បកែ បេ យៈ ែកវ សិរ                                                                                        Page 18
បជុំវ          សិស ពូែកគណិតវទ        ក់ទ១០
                                                                                     ី



                                                                ចំេលយ

 .    Lន.< នM< Q      មYពក:                ., ពប! Sប#, យ)ង,ន
                     1 1 1
               3=     + + ≥ 3 3 abc                    ⇔ abc ≥ 1
                     a b c
      Fង       x = 12 a , y = 12 b , z = 12 c

      8!f # q យ G6

                   x > 0, y > 0, z > 0
      ច! @                             ,                  យប>ក#5 x 9 + y 9 + z 9 ≥ x8 + y 8 + z 8
                                                              ?
                   xyz ≥ 1
      Lន.< នM< Q      មYពក:            .      9 ច! នBន

               x 9 + ... + x 9 + 1 ≥ 9 x8            (1)
               y 9 + ... + y 9 + 1 ≥ 9 y 8           (2)
               z 9 + ... + z 9 + 1 ≥ 9 z 8           (3)

       យ)ង      ន          x 8 + y 8 + z 8 ≥ 3 3 x 8 y 8 z 8 ≥ 3 (4)

      ប:កLងXន%ងLងXប               <Q        មYព (1), (2), (3), (4) យ)ង,ន8ទ}ផ8;C8                           g<       យប>ក#
                                                                                                                       ?

               x 9 + y 9 + z 9 ≥ x8 + y 8 + z 8

                                                      A = − x1 − x2 + x3 + x4 > 0
                                                     B = x x − x x − x x − x x − x x + x x > 0
                                                     
 .    ពQនQ     បព| ន}< Q    ម
                                                            1 2      1 3    1 4      2 3     2 4     3 4
                                                     
                                                     C = x1 x2 x3 + x1 x3 x4 − x1 x3 x4 − x2 x3 x4 > 0
                                                      D = x1 x2 x3 x4 > 0
                                                     
      Fង f ( x) = ( x − x1 )( x − x2 )( x + x3 )( x + x4 )

      គ.     ព*q       យ)ង,ន f ( x) = x 4 + Ax 2 + Bx 2 + Cx + D

             យប         មគ.            A, B, C , D > 0 *             ម       f ( x) = 0 មQន†ច      នH           x < Qជ?   នទ

      C:ច *        x1 , x2 ≤ 0 , ផ0.យពប! Sប#

      C:ច ន       បព|ន};C8                 គIមQន       នH




2012-11-17បកែ បេ យៈ ែកវ សិរ                                                                                               Page 19
បជុំវ       សិស ពូែកគណិតវទ       ក់ទ១០
                                                                          ី


+.    ច!នBន;C8        g< ក    នSង abc , ច! @ a, b, c ∈ ℕ 9)យ 1 ≤ a ≤ 9, 0 ≤ b, c ≤ 9

                             bca + cab
      Fមប! Sប#       abc =                    ⇔ 189a = 81b + 108c
                                 2
                ⇔ 7a = 3b + 4c ⇔ 7(a − b) = 4(c − b)                   (1) .

      Jញ,ន             4(c − b) ⋮ 7 (2)

                ⇒ c − b ⋮ 7 (3) .

             យ 0 ≤ b, c ≤ 9 * −9 ≤ c − b ≤ 9 (4)

      ព (3) នQង (4) Jញ,ន c − b = −7, 0, 7                 ពQ នQ    ប    ក

      ក        ទ p     c − b = −7 ⇔ b = c + 7 ≤ 9

                ⇒ c = 0, 1, 2 ⇒ b = 7,8,8 ⇒ a = 3, 4,5

                ⇒ abc = 370, 481, 592 .

      ក        ទ p     c−b = 0 ⇔ b = c

                ⇒ a = b = c ⇒ abc = 111, 222, ...., 999 .

      ក        ទ p     c−b = 7 ⇔ c =b+7 ≤9

                ⇒ b = 0,1,2 ⇒ c = 7,8, 9 ⇒ a = 4,5, 6 .

                ⇒ abc = 470, 581, 692 .

      C:ច ន        នJ!ងL       # 15 ច! នBន;C8         g< កគI

                       370, 481, 592, 470, 581, 692, 111, 222, 333, ..., 999

=.    4ប       5ព9.           $ កក-.ង ង"ង#ផQj
                                %                     O      ! R
                                                                                           α
      Fង α = AOB ( 00 < α < 1200 )                 ង# OH ⊥ AB , Jញ,ន AB = 2 HB = 2 R sin
                                                                                           2
                                                       3α
      C:ចW;C
          -          AC = 2 R sin α , AD = 2 R sin
                                                        2
                                  1             1      1
      ជ!នB     ច:8ប! Sប#                  =        +
                                      α       sin α sin 3α
                                sin
                                      2                  2

2012-11-17បកែ បេ យៈ ែកវ សិរ                                                                    Page 20
បជុំវ       សិស ពូែកគណិតវទ           ក់ទ១០
                                                                                ី


                                      3α      α             3α 
      C:ច *               sin α sin      − sin  sin α + sin    =0
                                       2      2              2 
                  1    α      5α  1     α      3α  1
                    cos − cos     −  cos − cos     − ( cos α − cos 2α ) = 0
                  2    2       2  2     2       2  2
                       3α                          5α 
       I           cos    + cos 2α  −  cos α + cos    =0
                        2                           2 
                        7α    α α
                  cos      sin sin = 0
                         4    4   2
      ច! @ 8ក^ខ              _ 0 < α < 1200 , យ)ង,ន

                        7α             7α             3000
                  cos      =0 ⇒           = 900 ⇒ α =
                         4              4              7
      C:ច ន ព9.                       ន ជងច!នBន 7
                                                     '()&



                        វ1     េសរ បឡងអូ              ពិចេវ ត
                                                      ំ                   ម)ក់ទី១០ េលកទី XII

                                                      វ           ទី៦

 .     a). គ                Q
                    3 ច!នBន<ជ?     ន;C8           នផ8ប: ក          )( 4       យប>ក# 5ផ8ប:កAនព ច! នBន
                                                                                 ?

             ក•     យក-.ង 3 ច!នBន * គI មQន :ច6ផ8គ.                         Aន 3 ច!នBន * ទ

       b). គ       * S = sin 390 + sin 690 + sin1830 + sin 2130

                                            1+ 3 x
 .     a).               យ     ម                          −1 = 0
                                         4x + 2 + x
       b). Fង x, y PងW6 ˆ
                      -  "                      #ម.!J!ងព ក-.ងព9.             នQ យ|   D1 នQង D1   យC%ង

      5 5x − 7 y = 0           កច! នBន ជង ប           # D1 , D2

+.                                    Q
             យប>ក#5 ច! @ គប# ច!នBនគ #<ជ?
                ?                                             ន n គ,ន

                  1   1    1             1
                    + 3 + 3 + ... +             <3
                  2 3. 2 4. 3       (1 + n).3 n


2012-11-17បកែ បេ យៈ ែកវ សិរ                                                                         Page 21
បជុំវ       សិស ពូែកគណិតវទ        ក់ទ១០
                                                                        ី


=.     គ                  ABC     ម,          ង# A         យC%ង5 L       :   ង# H ប        #    Qn

       o 8) ង"ង#$ %កក-.ង ប          #             *      ច: គ     * cos A
                                                '()&


                                                   ចំេលយ

 .     a). Fមប! Sប# គ         3 ច!នBនពQ      a, b, c នQង a + b + c = 4

           យមQន ធ")      , #8ក^            ទ: G, យ)ង        យប>ក#5
                                                               ?             a + b ≥ abc

      ព (a + b) 2 ≥ 4ab Jញ,ន (a + b + c) 2 ≥ 4(a + b)c

       ⇔ 16 ≥ 4(a + b)c ⇔ 16(a + b) ≥ 4(a + b)2 c ≥ 16abc .

       ⇔ a + b ≥ abc .

         មYព ក)          ន ព8 a = b = 1, c = 2

       b). យ)ង       ន

              S = 2sin 540 cos150 + 2sin1980 cos150 .

                 = 2cos150 ( sin 540 + sin1980 ) = 2 cos150 ( sin 540 − sin180 )

                 = 4cos150 cos360 sin180
                     4cos150 cos360 sin180 cos180 2cos150 cos360 sin 360
                 =                               =
                               cos180                    cos180
                     cos150 sin 720
                 =             0
                                    = cos150
                        sin 72
                      6+ 2
              S=
                       4
 .     a). 8ក^ខ       _ x≥0

         ម     ;C8          1 + 3 x − 4x − 2 + x = 0

       ⇔ 3 x − 2 + x = 4 x − 1 ⇔ 8 x − 2 = (4 x − 1) 3 x + 2 + x 
                                                     
                                                                  

                                            4 x − 1 = 0
       ⇔ (4 x − 1) 3 x + 2 + x − 2  = 0 ⇔ 
                                   
                                            3 x + 2 + x = 2
2012-11-17បកែ បេ យៈ ែកវ សិរ                                                                    Page 22
បជុំវ             សិស ពូែកគណិតវទ                    ក់ទ១០
                                                                                                        ី


                                             1
      * 4x − 1 = 0 ⇔                x=
                                             4
                                                                                             7−3 5
      * 3 x + 2 + x = 2 យ)ង                                  យ,នH                     x=
                                                                                               8
                                                            1      7−3 5
           ន-Q ‰ ន        ម                 នH      ព គI x = ; x =
                                                            4        8
       b). Fងច! នBន ជង ប                     #ព9.                    នQ យ|        D1 , D2 PងW-                       យ n នQ ង k

      8ក^ខ           _                   Q
                         n, k 6ច! នBនគ #<ជ?                       ន 9)យ 3 ≤ k ≤ n
                                                                                      (n − 2)π                                             (k − 2)π
          យ)ង,ន ˆ"             #ម.!នមB យk ប                      # D1 គI         x=                      9)យ ប              # D2 គI y =
                                                                                          n                                                    k
                                                                 5(n − 2)π 7(k − 2)π
                           5x − 7 y = 0 ⇔                                 =
                                                                     n         k
                ⇔ 5nk − 10k = 7nk − 14n .
                                                                     7n
                ⇔ 5k + nk = 7 n ⇔ k =
                                                                    n+5
                                     35
                ⇔ k =7−
                                    n+5
                           Q
            យ k 6ច! នBនគ #<ជ?                     ន * 35 ⋮ ( n + 5)

      C:ច * ( n + 5)               g<        )(ន%ង 1, 5, 7 I 35

      ;       យ k ≥ 3 ⇒ (n + 5) = 35

      C:ច ន n = 30 9)យ k = 6

+.                     Q
      ច! @ គប#ច!នBនគ #<ជ?                        ន k , យ)ង                   ន

                               1            1            3
                                                             k +1 − 3 k                                                 1
                                   −               =                             =
                           3
                               k        3
                                            k +1             3
                                                                 k (k + 1)            3
                                                                                          k . 3 k + 1.   (   3
                                                                                                                 (k + 1) 2 + 3 k (k + 1) + 3 k 2   )
                             1      1                  1                     1
      Jញ,ន                      −3      >                             =
                           3
                              k    k + 1 3 k . 3 k + 1. 3 3 (k + 1) 2        (
                                                                        3(1 + k ). 3 k           )
                                   1         1      1 
      C:ច ន                              < 3 3 − 3      
                           (1 + k ). 3 k     k     k +1 


2012-11-17បកែ បេ យៈ ែកវ សិរ                                                                                                                            Page 23
បជុំវ       សិស ពូែកគណិតវទ           ក់ទ១០
                                                                          ី


                       1   1   1              1              1   1   1           1 
      *!                 + 3 + 3 + ... +            < 3 1 − 3 + 3 − 3 + ... − 3      <3
                       2 3 2 4. 3        (n + 1). n
                                                 3
                                                              2   2   3         n +1 

=.    Fង O 6ផjQ        ង"ង#$ កក-.ង
                             %                      ,       ន   ! r 9)យ K 6ច!ន.ចក          8ប      # BC
                                                        A
      Fង x 6 ˆ"          #ម.! BHK ⇒ x = 900 −
                                                        2
      Fង y 6 ˆ"          #ម.! BOK , យ)ង,ន

                                          A         A                             A
            2 y = 1800 − B = 1800 −  900 −  = 900 +   ⇒               y = 450 +
                                          2         2                             4
                                          BK                                            BK
                   BHK     ន tan x =         9)យ                    BOK     ន tan y =      = 2 tan x
                                          2r                                             r
                       A               A          A
       ⇔     tan  450 +  = 2 tan  900 −  = 2cot g
                       4               2          2
                     A             A               A            A        A        A
             1 + tan     1 − tan 2              cos  + sin          cos 2 − sin 2
       ⇔             4 =           4      ⇔        4            4 =      4        4
                     A          A                  A            A        A     A
             1 − tan        tan                 cos − sin             sin .cos
                     4          4                  4            4        4     4
                                                2
                   A     A      A     A           A     A
       ⇔     sin     .cos =  cos − sin  = 1 − 2sin .cos
                   4     4      4     4           4     4
                   A     A 1      A 2
       ⇔     sin     .cos =  ⇔ sin =
                   4     4 3      2 3
                                                A    8 1
      ព * យ)ង,ន            cos A = 1 − 2sin 2     =1− =
                                                2    9 9
                                                '()&




2012-11-17បកែ បេ យៈ ែកវ សិរ                                                                            Page 24
បជុំវ      សិស ពូែកគណិតវទ                     ក់ទ១០
                                                                                                    ី



                        វ1              េសរ បឡងអូ                 ពិចេវ ត
                                                                  ំ                       ម)ក់ទី១០ េលកទី XII

                                                                  វ                 ទី៧

 .                     យ     ម           Zង                ម 8)       !ន.!ច!នBនពQ

              x 4 + 2006 x3 + 1006009 x 2 + x − 2 x + 2007 + 1004 = 0

 .    បˆញ5 ប) x1 , x2 6ប
        e                                               H     ប        #       ម      x 2 − 6 x + 1 = 0 * ច! @ គប# n∈ ℕ

      ច!នBន x1 + x2 6ច!នBនគ # មBយ;ចកមQ ន
             n    n
                                                                               ច#ន%ង 5

+.    ពQនQ   ប                      Q
                           ច!នBនពQ <ជ?                 ន a, b, c ផ0ង] #8ក^
                                                                      0                      _    2006ac + ab + bc = 2006

                                                                      2               2b 2               3
       ក !A8ធ! ប!ផ.             ប        #ក នŠម P =                             −                  +
                                                                  a 2 + 1 b 2 + 20062                  c2 + 1
=.     គ     ច .                @យ ABCD
                                 -                           ន,                : ចគI AB      ង"ង#មBយ             #Fម B នQ ង C ប/ ន%ង

       ជង AD               ង# E , ង"ង#មBយ                    #Fម A នQ ង D ប/ ន%ង ជង BC                           ង# F   ង"ង#J!ងព

       ន         #W-        ង#ព ច!ន.ច M នQ ង N                                  យប>ក# 5
                                                                                   ?                             J!ងព EMN នQ ង

       FMN             ន កmAផ0                 ) (W-
                                                                  '()&


                                                                       ចំេលយ
                                        2007
 .    8ក^ខ         _        x≥−                    (*)            ម                មម:8ន% ង
                                          2

              x 2 ( x 2 + 2.x.1003 + 10032 ) +
                                                              1
                                                              2
                                                                  (
                                                                2 x + 2007 − 2 x + 2007 + 1 = 0              )
                                1
                                               (                           )
                                                 2
       ⇔      x 2 ( x + 1003) 2 +  2 x + 2007 − 1 = 0
                                2
                                                                                          ( ផ0ង] # (*) )
                                                                                                 0
               x( x + 1003) = 0
              
       ⇔                             ⇔ x = −1003
               2 x + 2007 − 1 = 0
              

      C:ច ន H               ប       #    ម          គI x = −1003


2012-11-17បកែ បេ យៈ ែកវ សិរ                                                                                                      Page 25
បជុំវ        សិស ពូែកគណិតវទ               ក់ទ១០
                                                                                         ី


 .     + C! ប:ង យ)ង              យប>ក#
                                    ?

                 ច! @ គប# n ∈ ℕ , ច!នBន x1n + x2 6ច!នBនគ # (*)
                                               n



           !    ) ពQ ច! @ n = 0, n = 1, n = 2                    យ)ង,ន

                 x10 + x2 = 1 + 1 = 2 .
                        0



                 x1 + x1 = 6; x12 + x2 = ( x1 + x2 )2 − 2 x1 x2 = 62 − 2.1 = 34 .
                  1
                       2
                                     2



      4ប        5      !     )   (*) ពQ ច! @ n = k − 1                 ច! @ n = k , យ)ង,ន

       x1k + x2 = ( x1 + x2 ) ( x1k −1 + x2 −1 ) − x1 x2 ( x1k − 2 + x2 − 2 )
              k                           k                           k



                 = 6 ( x1k −1 + x2 −1 ) − ( x1k − 2 + x2 − 2 ) = 5 ( x1k −1 + x2 −1 ) + ( x1k −1 + x2 −1 ) − ( x1k − 2 + x2 − 2 ) (**)
                                 k                     k                       k                    k                     k



      C:ច * , ប) ( x1k −1 + x2 −1 ) នQង ( x1k − 2 + x2 − 2 ) 6ប
                             k                       k
                                                                                   ច! នBនគ # *               x1k + x2 6ច! នBនគ #
                                                                                                                    k



      ព * Fម< Q$ Lន.                    ន មគ
                                          B          Q   <QទO *           x1n + x2 6ច! នBនគ #ច! @ គប# n
                                                                                 n



       + „c:< យ)ង                យប>ក# x1n + x2 ;ចកមQន
                                    ?         n
                                                                            ច# ន%ង 5 Fម<Qធ                   យផ0.យព         ពQ

      4ប        5          នប       ច!នBនគ # ធម(6 Q n V/ ង                                x1n + x2 ;ចក
                                                                                                 n
                                                                                                                 ច#ន%ង 5

      Fង n0 6ច! នBនគ #ធម(6 Q : ចប!ផ. ;                             x1n0 + x2 0 ;ចក
                                                                           n
                                                                                           ច# ន%ង 5

      Fម (**) * ផ8Cក ( x1n0 −1 + x2 0 −1 ) − ( x1n0 − 2 + x2 0 − 2 ) ក•
                                  n                        n
                                                                                          g<;ចក        ច# ន%ង 5 ;C

      ជ!នB       k         យ n0 − 1 ក-.ង (**) យ)ង,ន

                                       (                  ) (                   ) (
                 x1n0 −1 + x2 0 −1 = 5 x1n0 − 2 + x2 0 − 2 + x1n0 − 2 + x2 0 − 2 − x1n0 − 3 + x2 0 − 3
                            n                      n                     n                     n
                                                                                                         )
      ព * Jញ,ន

                                       (             n
                                                          ) (              n
                                                                                  ) (            n
                                                                                                         
                 x1n0 − 3 + x2 0 − 3 = 5 x1n0 − 2 + x2 0 − 2 −  x1n0 −1 + x2 0 −1 − x1n0 − 2 + x2 0 − 2 
                             n
                                                                                                             )
      ក•       g<;ចក       ច#ន%ង 5 ;C         ក          ន ផ0.យព            4ប       5 n0 6ច! នBនគ # ធម(6 Q : ចប! ផ.

      ;C8 x1n0 + x2 0 ;ចក
                  n
                                        ច# ន%ង 5

      C:ច * , ប!Sប# x1n + x2 ;ចក
                           n
                                                    ច#ន%ង 5 គI មQន         ន

      C:ច ន          x1n + x2 ;ចកមQ ន
                            n
                                               ច# ន%ង 5 ច! @ គប# n




2012-11-17បកែ បេ យៈ ែកវ សិរ                                                                                                       Page 26
បជុំវ       សិស ពូែកគណិតវទ     ក់ទ១០
                                                                     ី


                                            ab   bc
+.    Fមប! Sប# យ)ងJញ,ន ac +                    +     = 1,
                                           2006 2006
           យ a, b, c > 0 *         ន A, B, C ∈ (0, π ) V/ ង         A+ B+C =π
                            A    B     B   C     C   A            ab   bc
       9)យ      យ tan         tan + tan tan + tan tan = 1 = ac +     +
                            2    2     2   2     2   2           2006 2006
                                A   b       B         C
       * ប) Fង a = tan            ;    = tan ; c = tan គI យ)ង,ន
                                2 2006      2         2
                          2                             2
              P=               −
                           A          1         3            A        B         C
                     tan 2 + 1            +          = 2cos 2 − 2sin 2 + 3cos 2
                           2          B         C            2        2         2
                                 tan 2 + 1 tan 2 + 1
                                       2         2
                                                 C          C      C    A−B
                = cos A + cos B + 3 − 3sin 2       = −3sin 2 + 2sin cos     +3
                                                 2          2      2     2
                            C         C 1      A− B    1    10
                ≤ −3sin 2     + 3sin 2 + cos 2      +3≤ +3=
                            2         2 3       2      3     3
                                             2
                     C 1       A−B
      ព*       3 sin 2 −   cos     ≥0
                         3      2 
                     C     A− B         C 1      A− B
       ⇔      2sin     cos      ≤ 3sin 2 + cos 2
                     2      2           2 3       2
                                                A− B
                                            cos 2 = 1              A = B
                                                                   
         > " = " ក)
          a                 ន8. F;                               ⇔  C 1
                                            3sin C = cos A − B     sin 2 = 3
                                                                    
                                            
                                                 2         2
                            1           2
      Jញ,ន           c=         ; a=      ; b = 1003 2
                          2 2          2
                          10         1        2
      C:ច ន max P =          ព8 c =     ; a=    ; b = 1003 2
                           3        2 2      2

=.    Fង      I = EF ∩ MN ; K = AD ∩ BC ; P = EF ∩ ( ADF ); Q = EF ∩ ( BCE )

              ( ADE ) = (O); ( BCE ) = (O ') .

       យ)ង,ន

              PK / (O) = KF 2 = KA.KD (1); PK / (O ') = KE 2 = KB.KC (2)


2012-11-17បកែ បេ យៈ ែកវ សិរ                                                         Page 27
Vnmo 30 4-2006-grade 10
Vnmo 30 4-2006-grade 10
Vnmo 30 4-2006-grade 10
Vnmo 30 4-2006-grade 10
Vnmo 30 4-2006-grade 10
Vnmo 30 4-2006-grade 10
Vnmo 30 4-2006-grade 10
Vnmo 30 4-2006-grade 10
Vnmo 30 4-2006-grade 10
Vnmo 30 4-2006-grade 10
Vnmo 30 4-2006-grade 10
Vnmo 30 4-2006-grade 10
Vnmo 30 4-2006-grade 10
Vnmo 30 4-2006-grade 10
Vnmo 30 4-2006-grade 10
Vnmo 30 4-2006-grade 10
Vnmo 30 4-2006-grade 10
Vnmo 30 4-2006-grade 10
Vnmo 30 4-2006-grade 10
Vnmo 30 4-2006-grade 10
Vnmo 30 4-2006-grade 10
Vnmo 30 4-2006-grade 10
Vnmo 30 4-2006-grade 10
Vnmo 30 4-2006-grade 10
Vnmo 30 4-2006-grade 10
Vnmo 30 4-2006-grade 10
Vnmo 30 4-2006-grade 10
Vnmo 30 4-2006-grade 10
Vnmo 30 4-2006-grade 10
Vnmo 30 4-2006-grade 10
Vnmo 30 4-2006-grade 10
Vnmo 30 4-2006-grade 10
Vnmo 30 4-2006-grade 10
Vnmo 30 4-2006-grade 10
Vnmo 30 4-2006-grade 10
Vnmo 30 4-2006-grade 10
Vnmo 30 4-2006-grade 10
Vnmo 30 4-2006-grade 10
Vnmo 30 4-2006-grade 10
Vnmo 30 4-2006-grade 10
Vnmo 30 4-2006-grade 10
Vnmo 30 4-2006-grade 10
Vnmo 30 4-2006-grade 10
Vnmo 30 4-2006-grade 10
Vnmo 30 4-2006-grade 10
Vnmo 30 4-2006-grade 10
Vnmo 30 4-2006-grade 10
Vnmo 30 4-2006-grade 10
Vnmo 30 4-2006-grade 10
Vnmo 30 4-2006-grade 10
Vnmo 30 4-2006-grade 10
Vnmo 30 4-2006-grade 10
Vnmo 30 4-2006-grade 10
Vnmo 30 4-2006-grade 10
Vnmo 30 4-2006-grade 10
Vnmo 30 4-2006-grade 10
Vnmo 30 4-2006-grade 10
Vnmo 30 4-2006-grade 10
Vnmo 30 4-2006-grade 10
Vnmo 30 4-2006-grade 10
Vnmo 30 4-2006-grade 10
Vnmo 30 4-2006-grade 10
Vnmo 30 4-2006-grade 10
Vnmo 30 4-2006-grade 10
Vnmo 30 4-2006-grade 10
Vnmo 30 4-2006-grade 10
Vnmo 30 4-2006-grade 10
Vnmo 30 4-2006-grade 10
Vnmo 30 4-2006-grade 10
Vnmo 30 4-2006-grade 10
Vnmo 30 4-2006-grade 10
Vnmo 30 4-2006-grade 10
Vnmo 30 4-2006-grade 10
Vnmo 30 4-2006-grade 10
Vnmo 30 4-2006-grade 10
Vnmo 30 4-2006-grade 10
Vnmo 30 4-2006-grade 10
Vnmo 30 4-2006-grade 10
Vnmo 30 4-2006-grade 10
Vnmo 30 4-2006-grade 10
Vnmo 30 4-2006-grade 10
Vnmo 30 4-2006-grade 10
Vnmo 30 4-2006-grade 10
Vnmo 30 4-2006-grade 10
Vnmo 30 4-2006-grade 10
Vnmo 30 4-2006-grade 10
Vnmo 30 4-2006-grade 10
Vnmo 30 4-2006-grade 10
Vnmo 30 4-2006-grade 10
Vnmo 30 4-2006-grade 10
Vnmo 30 4-2006-grade 10

More Related Content

Featured

AI Trends in Creative Operations 2024 by Artwork Flow.pdf
AI Trends in Creative Operations 2024 by Artwork Flow.pdfAI Trends in Creative Operations 2024 by Artwork Flow.pdf
AI Trends in Creative Operations 2024 by Artwork Flow.pdf
marketingartwork
 
Skeleton Culture Code
Skeleton Culture CodeSkeleton Culture Code
Skeleton Culture Code
Skeleton Technologies
 
PEPSICO Presentation to CAGNY Conference Feb 2024
PEPSICO Presentation to CAGNY Conference Feb 2024PEPSICO Presentation to CAGNY Conference Feb 2024
PEPSICO Presentation to CAGNY Conference Feb 2024
Neil Kimberley
 
Content Methodology: A Best Practices Report (Webinar)
Content Methodology: A Best Practices Report (Webinar)Content Methodology: A Best Practices Report (Webinar)
Content Methodology: A Best Practices Report (Webinar)
contently
 
How to Prepare For a Successful Job Search for 2024
How to Prepare For a Successful Job Search for 2024How to Prepare For a Successful Job Search for 2024
How to Prepare For a Successful Job Search for 2024
Albert Qian
 
Social Media Marketing Trends 2024 // The Global Indie Insights
Social Media Marketing Trends 2024 // The Global Indie InsightsSocial Media Marketing Trends 2024 // The Global Indie Insights
Social Media Marketing Trends 2024 // The Global Indie Insights
Kurio // The Social Media Age(ncy)
 
Trends In Paid Search: Navigating The Digital Landscape In 2024
Trends In Paid Search: Navigating The Digital Landscape In 2024Trends In Paid Search: Navigating The Digital Landscape In 2024
Trends In Paid Search: Navigating The Digital Landscape In 2024
Search Engine Journal
 
5 Public speaking tips from TED - Visualized summary
5 Public speaking tips from TED - Visualized summary5 Public speaking tips from TED - Visualized summary
5 Public speaking tips from TED - Visualized summary
SpeakerHub
 
ChatGPT and the Future of Work - Clark Boyd
ChatGPT and the Future of Work - Clark Boyd ChatGPT and the Future of Work - Clark Boyd
ChatGPT and the Future of Work - Clark Boyd
Clark Boyd
 
Getting into the tech field. what next
Getting into the tech field. what next Getting into the tech field. what next
Getting into the tech field. what next
Tessa Mero
 
Google's Just Not That Into You: Understanding Core Updates & Search Intent
Google's Just Not That Into You: Understanding Core Updates & Search IntentGoogle's Just Not That Into You: Understanding Core Updates & Search Intent
Google's Just Not That Into You: Understanding Core Updates & Search Intent
Lily Ray
 
How to have difficult conversations
How to have difficult conversations How to have difficult conversations
How to have difficult conversations
Rajiv Jayarajah, MAppComm, ACC
 
Introduction to Data Science
Introduction to Data ScienceIntroduction to Data Science
Introduction to Data Science
Christy Abraham Joy
 
Time Management & Productivity - Best Practices
Time Management & Productivity -  Best PracticesTime Management & Productivity -  Best Practices
Time Management & Productivity - Best Practices
Vit Horky
 
The six step guide to practical project management
The six step guide to practical project managementThe six step guide to practical project management
The six step guide to practical project management
MindGenius
 
Beginners Guide to TikTok for Search - Rachel Pearson - We are Tilt __ Bright...
Beginners Guide to TikTok for Search - Rachel Pearson - We are Tilt __ Bright...Beginners Guide to TikTok for Search - Rachel Pearson - We are Tilt __ Bright...
Beginners Guide to TikTok for Search - Rachel Pearson - We are Tilt __ Bright...
RachelPearson36
 
Unlocking the Power of ChatGPT and AI in Testing - A Real-World Look, present...
Unlocking the Power of ChatGPT and AI in Testing - A Real-World Look, present...Unlocking the Power of ChatGPT and AI in Testing - A Real-World Look, present...
Unlocking the Power of ChatGPT and AI in Testing - A Real-World Look, present...
Applitools
 
12 Ways to Increase Your Influence at Work
12 Ways to Increase Your Influence at Work12 Ways to Increase Your Influence at Work
12 Ways to Increase Your Influence at Work
GetSmarter
 
ChatGPT webinar slides
ChatGPT webinar slidesChatGPT webinar slides
ChatGPT webinar slides
Alireza Esmikhani
 
More than Just Lines on a Map: Best Practices for U.S Bike Routes
More than Just Lines on a Map: Best Practices for U.S Bike RoutesMore than Just Lines on a Map: Best Practices for U.S Bike Routes
More than Just Lines on a Map: Best Practices for U.S Bike Routes
Project for Public Spaces & National Center for Biking and Walking
 

Featured (20)

AI Trends in Creative Operations 2024 by Artwork Flow.pdf
AI Trends in Creative Operations 2024 by Artwork Flow.pdfAI Trends in Creative Operations 2024 by Artwork Flow.pdf
AI Trends in Creative Operations 2024 by Artwork Flow.pdf
 
Skeleton Culture Code
Skeleton Culture CodeSkeleton Culture Code
Skeleton Culture Code
 
PEPSICO Presentation to CAGNY Conference Feb 2024
PEPSICO Presentation to CAGNY Conference Feb 2024PEPSICO Presentation to CAGNY Conference Feb 2024
PEPSICO Presentation to CAGNY Conference Feb 2024
 
Content Methodology: A Best Practices Report (Webinar)
Content Methodology: A Best Practices Report (Webinar)Content Methodology: A Best Practices Report (Webinar)
Content Methodology: A Best Practices Report (Webinar)
 
How to Prepare For a Successful Job Search for 2024
How to Prepare For a Successful Job Search for 2024How to Prepare For a Successful Job Search for 2024
How to Prepare For a Successful Job Search for 2024
 
Social Media Marketing Trends 2024 // The Global Indie Insights
Social Media Marketing Trends 2024 // The Global Indie InsightsSocial Media Marketing Trends 2024 // The Global Indie Insights
Social Media Marketing Trends 2024 // The Global Indie Insights
 
Trends In Paid Search: Navigating The Digital Landscape In 2024
Trends In Paid Search: Navigating The Digital Landscape In 2024Trends In Paid Search: Navigating The Digital Landscape In 2024
Trends In Paid Search: Navigating The Digital Landscape In 2024
 
5 Public speaking tips from TED - Visualized summary
5 Public speaking tips from TED - Visualized summary5 Public speaking tips from TED - Visualized summary
5 Public speaking tips from TED - Visualized summary
 
ChatGPT and the Future of Work - Clark Boyd
ChatGPT and the Future of Work - Clark Boyd ChatGPT and the Future of Work - Clark Boyd
ChatGPT and the Future of Work - Clark Boyd
 
Getting into the tech field. what next
Getting into the tech field. what next Getting into the tech field. what next
Getting into the tech field. what next
 
Google's Just Not That Into You: Understanding Core Updates & Search Intent
Google's Just Not That Into You: Understanding Core Updates & Search IntentGoogle's Just Not That Into You: Understanding Core Updates & Search Intent
Google's Just Not That Into You: Understanding Core Updates & Search Intent
 
How to have difficult conversations
How to have difficult conversations How to have difficult conversations
How to have difficult conversations
 
Introduction to Data Science
Introduction to Data ScienceIntroduction to Data Science
Introduction to Data Science
 
Time Management & Productivity - Best Practices
Time Management & Productivity -  Best PracticesTime Management & Productivity -  Best Practices
Time Management & Productivity - Best Practices
 
The six step guide to practical project management
The six step guide to practical project managementThe six step guide to practical project management
The six step guide to practical project management
 
Beginners Guide to TikTok for Search - Rachel Pearson - We are Tilt __ Bright...
Beginners Guide to TikTok for Search - Rachel Pearson - We are Tilt __ Bright...Beginners Guide to TikTok for Search - Rachel Pearson - We are Tilt __ Bright...
Beginners Guide to TikTok for Search - Rachel Pearson - We are Tilt __ Bright...
 
Unlocking the Power of ChatGPT and AI in Testing - A Real-World Look, present...
Unlocking the Power of ChatGPT and AI in Testing - A Real-World Look, present...Unlocking the Power of ChatGPT and AI in Testing - A Real-World Look, present...
Unlocking the Power of ChatGPT and AI in Testing - A Real-World Look, present...
 
12 Ways to Increase Your Influence at Work
12 Ways to Increase Your Influence at Work12 Ways to Increase Your Influence at Work
12 Ways to Increase Your Influence at Work
 
ChatGPT webinar slides
ChatGPT webinar slidesChatGPT webinar slides
ChatGPT webinar slides
 
More than Just Lines on a Map: Best Practices for U.S Bike Routes
More than Just Lines on a Map: Best Practices for U.S Bike RoutesMore than Just Lines on a Map: Best Practices for U.S Bike Routes
More than Just Lines on a Map: Best Practices for U.S Bike Routes
 

Vnmo 30 4-2006-grade 10

  • 1. បជុំវ សិស ពូែកគណិតវទ ក់ទី១០ បកែ បេ យ ែកវ សិរ"
  • 2. បជុំវ សិស ពូែកគណិតវទ ក់ទ១០ ី វ អូ ពិក បៃពណីេវ ត ំ មេលកទី XII ំ ២០០៦ មុខ វ$ៈ គណិត វទ()ក់ទី ១០ រយៈេពល ១៨០/ទី % . យ ម ( x + 1) x 2 − 2 x + 3 = x 2 + 1 . គ ABC នប ជង BC = a, CA = b, AB = c, B = 2 A, C = 4 A , !  1 1 1  ង"ង#$ %ក ' )(ន%ង R គ * T = R2  2 + 2 + 2  a b c  +. គ ABC ម, ង# A ង"ង#$ %កក-.ង ABC ប/ ជង AB ង# T , ប* # CT 0 # ង"ង# ង# K ផ2ងព T 4ប 5 K 6ច!ន.ចក 8 CT 9)យ CT = 6 2 ច: គ * ប;<ងប ជង ប # ABC =. យប>ក#5 ច! @ គប# !A86ច!នBនគ # ប ? # m, នច!នBនគ # n C)មD n3 − 11n 2 − 87 n + m ;ចក ច# ន%ង 191 E. គ a , b, c > 0 យប>ក# 5 ? a4 b4 c4 + + ≤1 a 4 +3 (a 6 +b 6 )( a 3 +c ) 3 2 b 4 +3 (b 6 +c 6 )( b 3 +a ) 3 2 c 4 +3 (c 6 +a 6 )( c 3 +b ) 3 2 '()& ចំេលយ . យ ម ( x + 1) x 2 − 2 x + 3 = x 2 + 1 (1) ;Cនក! # ℝ Fង x 2 − 2 x + 3 = t ច! @ t ≥ 2 ព8 * ម (1) G6 ( x + 1)t = x 2 + 1 (2) (2) ⇔ x 2 − 2 x + 3 − ( x + 1)t + 2( x − 1) = 0 2012-11-17បកែ បេ យៈ ែកវ សិរ Page 1
  • 3. បជុំវ សិស ពូែកគណិតវទ ក់ទ១០ ី t = 2 ⇔ t 2 − ( x + 1)t + 2( x − 1) = 0 ⇔  t = x − 1 ច! @ t = 2 , យ)ង,ន x2 − 2 x + 3 = 2 ⇔ x2 − 2 x + 3 = 4 x = 1− 2 ⇔ x2 − 2x − 1 = 0 ⇔  x = 1+ 2  ច! @ t = x − 1 , យ)ង,ន x2 − 2 x + 3 = x − 1 x −1 ≥ 0 x ≥ 1 ⇔  2 ⇔  ⇔ x ∈∅  x − 2 x + 3 = ( x − 1) 3 = 1 2 C:ច ន !ន.!H ប # ម { គI 1 − 2;1 + 2 } a b c . យ)ង ន = = = 2R sin A sin B sin C R 1 R 1 R 1 ⇒ = , = , = a 2sin A b 2sin B c 2sin C 1 1 1 1  ⇒ S=  2 + +  4  sin A sin B sin 2 C  2 = 1 4 ( 3 + cot g 2 A + cot g 2 B + cot g 2C ) ក-.ង ∆ABC ន cot gA.cot gB + cot gB.cot gC + cot gC.cot gA = 1 cot g 2α − 1 9)យ យ)ង ន cot g 2α = , ក-.ង * cot g 2α = 1 + 2cot gα .cot g 2α 2cot gα 1 ⇒ S= 3 + 3 + 2 ( cot gA.cot g 2 A + cot gB.cot g 2 B + cot gC.cot g 2C )  4  1 1 = 6 + 2 ( cot gA.cot gB + cot gB.cot gC + cot gC.cot gA )  = (6 + 2) = 2   4 4 +. K 6ច! ន.ចក 8ប # CT 9)យ L 6ច! ន.ចប/ ប # ង"ង# Gន% ង ជង BC 1 * CK = CT (*) Jញ,ន L 6ច! ន.ចក 8 BC , 2 1 CL2 = CK .CT = CT 2 , 2 2012-11-17បកែ បេ យៈ ែកវ សិរ Page 2
  • 4. បជុំវ សិស ពូែកគណិតវទ ក់ទ១០ ី I a 2 / 4 = 36 , I a = 12 (1) Lន.< នM ទ% បទក: .ន. ក-.ង BCT , យ)ង,ន CT 2 = BT 2 + BC 2 − 2 BT .BC.cos B ⇔ 72 = a 2 / 4 + a 2 − 144.cos B 3 ⇔ cos B = (2), Fង (1) 2 មO/ ង ទP , Lន.< នM ទ% បទក: .ន. ក-.ង ABC , យ)ង,ន b2 = c 2 + a 2 − 2ca.cos B ⇔ cos B = a / 2b (3) ព (1), (2), នQង (3) , យ)ង,ន (a, b, c) = (12,8,8) =. Fង P( x) = x 3 − 11x 2 − 87 x + m យ)ង,ន P( x) ≡ ( x + a)3 + b (mod 191) ⇔ x3 + 3ax 2 + 3a 2 x + a3 + b ≡ x3 − 11x 2 − 87 x + m (mod 191) 3a ≡ − 11 (mod 191) (1)  2 ⇔ 3a ≡ − 87(mod 191) (2) b ≡ m − a 3 (mod 191) (3)  (1) ⇔ 3a ≡ 180 (mod 191) ⇔ a ≡ 60 (mod 191) ⇒ 3a 2 ≡ − 87(mod 191) C:ច ន ∀m ∈ ℤ , នច!នBនគ # a, b C) មD P( x) ≡ ( x + a)3 + b (mod 191) ងR ម) 8 191 6ច! នBន;C8 នSង 191 = 3k + 2 P(i) ≡ P( j ) (mod 191) ⇒ (i + a)3 ≡ ( j + a)3 (mod 191) Fង u = i + a , v = j + a , * u 3 ≡ v3 (mod 191) ⇒ u 3k ≡ v3k (mod 191) u 3k v 2 ≡ v3k + 2 ≡ v191 (mod 191) ≡ v (mod 191) ( ទ% បទ Ferma ) (4) ⇒ v2 ≡ u 3k v3 ≡ u 3k + 3 (mod 191) ⇒ u 3k v2 ≡ u 3k .u 3k + 3 ≡ u 3k +1.u 3k + 2 ≡ u 3k +1.u191 ≡ u 3k +1.u (mod 191) 2012-11-17បកែ បេ យៈ ែកវ សិរ Page 3
  • 5. បជុំវ សិស ពូែកគណិតវទ ក់ទ១០ ី ≡ u 3k + 2 ≡ u191 ≡ u (mod 191) (5) ព (4) នQង (5) Jញ,ន u ≡ v (mod 191) ⇒ i ≡ j (mod 191) C:ច ន ប) ∀i, j ∈ {1, 2, ... , 191} : i ≠ j (mod 191) * P(i) ≠ P( j ) (mod 191) Jញ,ន ន n ∈ {1, 2, ..., 191} V/ ង P(n) ≡ 191(mod 191) , I P (n) ⋮ 191 C:ច ន ច! @ គប# !A8គ # ប # m, ; ង នច!នBនគ # n C)មD P (n) ⋮ 191 (a + b 6 )( a 3 + c3 ) = (a + b6 )( a 6 + 2a 3c3 + c 6 ) = (a + b 6 )( a 6 + 2a 3c 3 + c 6 ) 2 E. 3 6 3 6 3 6 = 3 (a 12 + 2a3b6c3 ) + ( 2a 9c3 + b6c 6 ) + a 6b6 + a 6c 6 (a b + a 2 c 2 ) = a 2b 2 + a 2 c 2 3 ≥ 3 a 6b 6 + 3a 6b 4c 2 + 2a 6b 2c 4 + a 6c 6 = 3 2 2 a4 a4 a2 Jញ,ន ≤ = 2 a 4 + 3 ( a 6 + b 6 )( a 3 + c3 ) 2 a 4 + a 2b 2 + a 2 c 2 a + b 2 + c 2 b4 b2 C:ចW;C - ≤ b 4 + 3 ( b6 + c 6 )( b3 + a 3 ) 2 a 2 + b2 + c 2 c4 c2 ≤ c 4 + 3 ( a 6 + b6 )( c3 + b3 ) 2 a2 + b2 + c2 ប:កLងXន%ងLងXAន មYពZង 8), យ)ង,ន a4 b4 c4 + + ≤1 . a + 4 3 (a 6 +b 6 )( a 3 +c ) 3 2 b + 4 3 (b 6 +c 6 )( b 3 +a 3 2 ) c + 4 3 (a 6 +b 6 )( c 3 +b ) 3 2 '()& 2012-11-17បកែ បេ យៈ ែកវ សិរ Page 4
  • 6. បជុំវ សិស ពូែកគណិតវទ ក់ទ១០ ី វ1 េសរ បឡងអូ ពិចេវ ត ំ ម)ក់ទី១០ េលកទី XII វ ទី១ 1 . យ ម − x 1 − x2 = 1 − 2 x2 (1) 2 . ពQនQ ប Q ច!នBនពQ មQ នL<ជ? ន ; ប ប[8 a1 , a2 , ..., a9 មQនផ8ប:ក )( 1 ( ) Fង S K = ak + ak +1 + ak + 2 + ak +3 k = 1;6 , Fង M = max {S1 , S2 , S3 , S4 , S5 , S6 } ច: ក! # !A8 : ចប! ផ. ប # M +. ក គប#Lន. គមនM f : ℝ → ℝ ផ0ង] # 8ក^ 0 _ ( ) ( ) f x3 − y + 2 y 3 f 2 ( x ) + y 2 = f ( y + f ( x ) ) ∀x, y ∈ ℝ (1) A B 1 =. គ ABC ផ0ង] # tan 0 tan = យប>ក# 5 8ក^ ? _ $!,ច# 2 2 2 A B C 1 នQង គប# Wន# C) មD ABC ;កងគI sin sin sin = 2 2 2 10 '()& ចំេលយ ( ) 2 1 1 . − x 1 − x2 = 1 − x2 − x 2 2 8ក^ខ _ C) មD ម (1) ក! # 1 − x2 ≥ 0 ⇔ x ≤1 ព8 * 1 (1) ⇔ 1 − x2 − x = 1 − 2 x2 2 1 ម នH ⇔ 1 − 2 x2 ≥ 0 ⇔ x ≤ 2 យ)ង ន 1 − 2 x2 ≥ 0 ⇒ 1 − x 2 ≥ x C: ច * (2) ⇔ 1 − x2 − x = 2 ( 1 − x2 + x )( 1 − x2 − x ) 2012-11-17បកែ បេ យៈ ែកវ សិរ Page 5
  • 7. បជុំវ សិស ពូែកគណិតវទ ក់ទ១០ ី  1 − x2 = x ⇔  (  2 1 − x2 + x = 1  )  1  x ≥ 0 x = 2   2  x = 1− x = x 2 2  ⇔  1 2 ⇔  x ≤ ⇔   1  2  2− 6  1 − x2 = −x  x =  2  2 1  4   2 x − 2.x − 2 = 0  C:ច ន ម (1) នH ព x1 = 2 2 ; x2 = 1 4 ( 2− 6 ) . M = max {S1 , S 2 , S3 , S 4 , S5 , S 6 } 9 ⇒ 12 M ≥ 4S1 + S 2 + S3 + S 4 + S5 + S6 = 4∑ ai + a2 + 2a3 + 3a4 + 3a6 + 2a7 + a8 ≥ 4 i =1 9 1 ( @ ∑ a = 1; a i =1 i ≥ 0, i = 1;9 ) ⇒ M ≥ 3  1 a1 = a5 = a9 = > " = " ក) a ន ព8  3 a2 = a3 = a4 = a6 = a7 = a8 = 0  1 ច! 8)យ M min = 3 +. ជ!នB y = x3 ច: 8 (1) យ)ង,ន f (0) + 2 x 3 ( 3 f 2 ( x) + x 6 ) = f ( x 3 + f ( x) ) (2) ជ!នB y = − f ( x) ច:8 (1) , Jញ,ន f ( x 3 + f ( x ) ) − 2 f ( x ) ( 3 f 2 ( x ) + f 2 ( x ) ) = f (0) (3) ព (2) នQង (3) ⇒ 4 f 3 ( x) − 3 f 2 ( x).x3 − x9 = 0 ; ∀x ⇒ ( f ( x) − x )( 4 f 3 2 ( x ) + x 3 f ( x ) + x 6 ) = 0 ; ∀x (4) ឃ)ញ5 4 f 2 ( x) + x3. f ( x) + x6 > 0 ; ∀x ≠ 0 ព (4) យ)ង,ន f ( x) = x3 ; ∀x 2012-11-17បកែ បេ យៈ ែកវ សិរ Page 6
  • 8. បជុំវ សិស ពូែកគណិតវទ ក់ទ១០ ី កជ!នB Q ច:8 c)ង<ញ, ផ0ង] #8ក^ខ 0 _ បdន ច! 8)យ f ( x) = x 3 A B =. យ)ង ន S = p ( p − a ) tan = p( p − b) tan = p ( p − a )( p − b)( p − c) 2 2 A B S2 p−c a+b−c ⇒ tan tan = 2 = = 2 2 p ( p − a)( p − b) p a+b+c A B 1 tan tan = ⇔ 2(a + b) − 2c = a + b + c ⇔ a + b = 3c (1) 2 2 2 abc r មO/ ង ទP S = pr = ⇒ p.abc. = S 2 = p( p − a )( p − b)( p − c) (2) 4R 4R  p = 2c r (1) ⇔  ព (2) Jញ,ន ab = p 2 − ( a + b ) p + ab p −c = c 4R  r  ⇒ 1 −  ab = 2c 2 (3)  4R  A B C 1 r 1 Fមប! Sប# sin sin sin = ⇒ = 2 2 2 10 4 R 10 20 2 ព (3) យ)ង,ន ab = c , មWន% ង (1) យ)ងJញ,ន a នQ ង b 6ប B - H ប # 9 20 2 5 4 ម t 2 − 3ct + c = 0; t1 = c; t2 = c 9 3 3  5 a = 3 c  យក a ≥ b , យ)ង,ន  b = 4 c   3 2 2 4  5  ⇒ b + c =  c  + c2 =  c  = a2 2 2 ⇒ ∆ABC ;កង ង# A 3  3  ផ0.យមក< Qញ, 4ប 5 ∆ABC ;កង ង# A យ)ង,ន  a = 2 R a = 2 R a = 2 R  2   a = b + c (1) ⇒ (3c − b) 2 = b 2 + c 2 ⇒ b = 4 r 2 2  1 b = 4r ( p = 2c) c = 3r  S = bc = pr    2 2012-11-17បកែ បេ យៈ ែកវ សិរ Page 7
  • 9. បជុំវ សិស ពូែកគណិតវទ ក់ទ១០ ី a = 2 R r 2 A B C 2 ព (1) យ)ង,ន  ⇒ 5r = 2 R ⇒ = ⇒ 4sin sin sin = a = 5r R 5 2 2 2 5 A B C 1 ⇒ sin sin sin = 2 2 2 10 (ប> ប e #8!f # g<,ន យប>ក#) ? '()& វ1 េសរ បឡងអូ ពិចេវ ត ំ ម)ក់ទី១០ េលកទី XII វ ទី២ x 2004 + 16 . គ <Q ម a + 1 − x 2006 + ≤ 2 x 2004 + 16 a + 1 − x 2006 កច! នBនពQ a ធ! ប!ផ. C)មD <Q ម នH ; ព ប/. i . គ m, n 6ព ច!នBនគ #< Qជ? ន យ ម 1 1 sin 2 n x + = cos 2n + sin 2 m +1 x cos 2m +1 x +. គ I 6ផjQ ង"ង#$ កក-.ង % មQន ម, ABC M 6ច!ន.ចក 8 IC , N 6ច!ន.ចក 8ប # AB J 6ច!ន.ចក 8ប # MN Fង x, y , z 6ប* # 0 ប PងW- #Fម A, B, C 9)យប* # នមBយk;ចកប Q 0 ABC 6ព ;ផ-ក )W ( - យប>ក# 5 4 ប* # x, y , z នQង IJ ? 0 #Fមច!ន.ច BមWមBយ - =. គ Q n 6ច!នBនគ #<ជ? ន ពQ នQ FSង 2n ជB CកនQង 2n ជB ឈ បm នមB យk ន ច! នBនមB យច:8 ;C8ច! នBន * Qn oក-.ង { } !ន.! 1, 2, 3, ..., 4 n 2 , ប m ព ផ2ងW- ច!នBន ផ2ងW- កច!នBន N ធ!ប!ផ. ;C8 ន8ក^ p ច! @ គប# បPប ច! នBនC: ចZង 8) នជB CកមB យ ជB ឈ មB យ ;C8 o 8) I ជB Cក IជB ឈ * នព ច! នBន p, q ផ0ង] # | p − q | ≥ N 0 '()& 2012-11-17បកែ បេ យៈ ែកវ សិរ Page 8
  • 10. បជុំវ សិស ពូែកគណិតវទ ក់ទ១០ ី ចំេលយ ( ) 2 a + 1 − x 2006 − x 2004 + 16 . c)ង< Qញ ≤ 0 ច! @ x 2 ≤ 1 a + 1− x 2006 ប) a < 0 < Q ម a + 1 − x 2006 < 0 នច! 8)យSប# មQនL # ប) a ≥ 0 យ)ង,ន ម a = 16 + x 2004 − 1 − x 2006 Lន. គមនM oLងXZង ! 6Lន. គមនM គ: Lន. គមនM ក) ន 8) [0;1] C:ច * ម នH ព8 a Qn oច * 3, 17  , Jញ,ន !A8ធ! ប!ផ. q   ប # a គI 17 , ព8 * H J!ងព គI x = −1, x = 1 . Fង u = sin x, v = cos x; u.v ≠ 0 9)យ u 2 + v 2 = 1 1 1 យ)ង,ន ម u 2n + 2 m +1 = v2n + 2 m +1 u v ប) uv < 0 LងX ^ ង : ច6ង 1 , LងX ^ ង ទP ធ!6ង 1 , មQ ន ផ0ង] # 0 1 Lន. គមនM y = x 2 n + 2 m +1 ច. o 8) [−1;0) C:ច ន ប) u, v < 0 ; u ≠ v គIមQន ផ0ង] # 0 x យ)ង,ន ( u 2 n − v 2 n ) ( u.v ) 2 m +1 ពQនQ u, v > 0; u ≠ v = u 2 m +1 − v 2 m +1 ( ) I u 2 n −1 + v 2 n −1 + uv ( u 2 n − 3 + v 2 n − 3 ) + ... + (uv) n −1 (u + v) (uv) 2 m +1 = u 2 m + v 2 m + u 2 m −1v + ... n−2 n−2 2 m +1 2 m +1  1  1 2 1  1   1  1 1 2m LងXZង ឆ"ង :ច6ង 1 + +   + ... +   +      < 2  =   2  2   2  2   2    2 2 m −1 1 LងXZង !ធ!6ង u 2 m + v 2 m ≥   C: ច * LងXZង ! : ច6ងLងXZង ឆ"ង 2 π C:ច ន sin x = cos x I x = + kπ 4 +. Lemma: ABC , Fង A ', B ', C ' PងW6ច!ន.ចក - 8ប #ប ជង BC , CA, AB ប ប* # 0 #Fមក!ព:8នមB យ ប # A ' B ' C ' 9)យ;ចកប Q A ' B ' C ' 6ព ប ព"W- ង#ច!ន.ច;C86ផjQ ង"ង#$ %កក-.ង ABC 2012-11-17បកែ បេ យៈ ែកវ សិរ Page 9
  • 11. បជុំវ សិស ពូែកគណិតវទ ក់ទ១០ ី ពQ 6C:ច ន : Fង M 6ច!ន.ច Qn o 8) B ' C ' V/ ង A ' M ;ចកប Q A ' B ' C ' 6ព * MC ' = p '− b ' 9)យ MB ' = p '− c ' ;C8 p ' = (a '+ b '+ c ') / 2 ( p '− c ') A′C ′ + ( p′ − b′) A′B′ ( p − c) A′C ′ + ( p − b) A′B′ Jញ,ន A′M = = = a' a A ( p − c) AC + ( p − c ) AB =− 2a មO/ ង ទP , យ)ង ន C' M B' I 2 p A′I = a A′A + b A′B + c A′C C ( ) B A' a AB + AC + (b − c) BC =− = −( p − b) AB − ( p − c) AC 2 C:ច ន A ', I , M # ង# ជB W- cប#មក8!f #< Qញ Fង G 6ទ បជ.! ទ!នង# ABC យ J 6ទ បជ.!ទ!ងន# ប #បBនច! ន.ច I , A, B, C * I , G, J ង# ង# ជB W- Fមចxប# ប!;8ង$!ងផjQ G Fមផ8 ធPប −1 / 2 ប!;8ង A G6 A ', B G6 B ', C G 6 C' ប!;8ងប* # J!ងប x, y , z G6ប* #J!ងប (Fម Lemma) ប 0 0 ព"W- ង# I ប:ក BមJ!ង I , G, J # ង# ជB W Jញ,ន x, y, z, IJ - # Fមច!ន.ច មWមB យ B - =. ពQនQ បPប PបC:ចZង ម 2n 2 − n + 1 ... ... 2n 2 ... 4n 2 . . . 2n + 1 n +1 2n 2n 2 + n + 1 ... 2n 2 + 2n 1 2 ... n 2n 2 + 1 ... 2n2 + n 2012-11-17បកែ បេ យៈ ែកវ សិរ Page 10
  • 12. បជុំវ សិស ពូែកគណិតវទ ក់ទ១០ ី ប) ន i, j o 8)ជB CកV/ ង i − j ≥ N * 2n 2 + n − 1 ≥ N ប) o 8)ជB ឈ * 2n 2 − n ≥ i − j ≥ N C:ច ន N ≤ 2n 2 + n − 1 ពQនQ N = 2n 2 + n − 1 Fង A = {1, 2, ..., n 2 − n + 1} ; B = {3n 2 , 3n 2 + 2, ..., 4n 2 } ច! @ គប# i Qn oក-.ង A, j Qn oក-.ង B , យ)ង,ន i − j ≥ 3n 2 − ( n 2 − n + 1) = 2n 2 + n − 1 (**) ច! @ ជB Cក ;C8 នផ0.កd . ប # A , យ)ង y56ជB C ប ភទទ , ជB ឈ ;C8 នផ0.កd . ប # A g<,ន y56ជB ឈ ប ភទទ ច! @ ជB Cក ;C8 នផ0.កធ d . ប # B , យ)ង y56ជB Cក ប ភទទ , ច! @ ជB ឈ ;C8 នផ0.កd . ប # B យ)ង y56ជB ឈ ប ភទទ Fង p, q PងW 6ច!នBនជB Cក នQ ងជB ឈ ប ភទទ - * p.q ≥ n2 − n + 1 , C:ច * p + q ≥ 2 pq ≥ 4n 2 − 4n + 4 > 2n − 1 Fង r , s PងW 6ច!នBនជB C , នQ ងជB ឈ ប ភទទ - * r.s ≥ n 2 + 1 , C:ច * r + s ≥ 2 rs ≥ 4n 2 + 4 > 2n C:ច ន p + q + r + s ≥ 4n + 1 , Jញ,ន នជB Cក ជB ឈ ;C86 ប ភទទ ផង នQ ង6 I ប ភទទ ផង ព (**) Jញ,ន !A8ធ!ប!ផ. ប # N គIp 2n 2 + n − 1 '()& 2012-11-17បកែ បេ យៈ ែកវ សិរ Page 11
  • 13. បជុំវ សិស ពូែកគណិតវទ ក់ទ១០ ី វ1 េសរ បឡងអូ ពិចេវ ត ំ ម)ក់ទី១០ េលកទី XII វ ទី៣ . ក គប# !A8 a, b, c, d , e ∈ [0,1] C)មD a b c d e A= + + + + =4 1 + bcde 1 + cdea 1 + deab 1 + eabc 1 + abcd . គ f ( x) = ax3 + bx 2 + cx + d V/ ង f ( x) ≤ 1, ∀x ∈ [ −1,1] (1) កច! នBន ថ k : ចប! ផ. C) មD 3ax 2 + 2bx + c ≤ k , ∀x ∈ [ −1,1], ∀f ផ0ង] # (1) 0 +. ក-.ងបqង#, គ ម| ង2 ABC ផjQ O ប* # ( d ) < Q8ជ.!<ញ O 0 Q #ប ប* # 0 1 1 1 BC , CA, AB PងW- ង# M , N , P យប>ក# 5 T = ? + + OM 4 ON 4 OP 4 គIមQន; ប ប[8 =. យប>ក#5 ច! @ គប# !A8គ # ប ? # m, នច! នBនគ # n C)មD n3 − 11n 2 − 87 n + m ;ចក ច# ន%ង 191 '()& ចំេលយ . យមQន ធ") , #បង# 8ក^ ទ: G 4ប 5 a ≤ b ≤ c ≤ d ≤ e (*) a b c d e a+b+c+d +e ព8 * A≤ + + + + = 1 + abcde 1 + abcde 1 + abcde 1 + abcde 1 + abcde 1 + abcde យ a, b, c, d , e ∈ [ 0,1] * : (1 − abc)(1 − de) + (1 − ab)(1 − c) + (1 − d )(1 − e) + (1 − b)(1 − a) ≥ 0 (1) ⇒ a + b + c + d + e ≤ 4 + abcde ≤ 4(1 + abcde) (2) . C:ច * A≤4 ប) A = 4 * មYព ក) ន o (1) នQង (2) Jញ,ន 2012-11-17បកែ បេ យៈ ែកវ សិរ Page 12
  • 14. បជុំវ សិស ពូែកគណិតវទ ក់ទ១០ ី abcde = 0 ⇒ a = 0 de = 1   a = 0 c = 1 ⇒  d = 1 ∨ e = 1 b = c = d = e = 1  b = 1  0 Q ផ0ង] # c)ង<ញ ព8 a = 0, b = c = d = e = 1 * A=4 C:ច ន ប បព|ន} !A8;C8 g< ក (a, b, c, d , e) 6ប ច!~ #ប # (0,1,1,1,1) . Fង A = f (−1), B = f (−1 / 2), C = f (1 / 2), D = f (1) * 2 4 4 2 2 2 2 2 a = − A + B − C + D, b= A− B − C + D 3 3 3 3 3 3 3 3 A 4 4 D A 2 2 D c= − B+ C− , d =− + B+ C− 6 3 3 6 6 3 3 6 A 4B h( x) = 3ax 2 + 2bx + c = − (12 x 2 − 8 x − 1) + (3 x 2 − x − 1) − 6 3 4C D (3 x 2 + x − 1) + (12 x 2 + 8 x − 1) 3 6 Fមប! Sប# Jញ,ន A , B , C , D ≤ 1 , C:ច * , ប) ∀x ∈ [ −1,1] គ,ន 1 4 4 1 h( x ) ≤ 12 x 2 − 8 x − 1 + 3 x 2 − x − 1 + 3x 2 + x − 1 + 12 x 2 + 8 x − 1 6 3 3 6 យ A + B = max ( A − B , A + B ) * ច! @ ∀x ∈ [ −1,1] យ)ង,ន ( 12 x 2 + 8 x − 1 + 12 x 2 − 8 x − 1 = max 16 x , 24 x 2 − 2 ≤ 22 ) ( 3 x 2 + x − 1 + 3 x 2 − x − 1 = max 2 x , 6 x 2 − 2 ≤ 4 ) 22 16 ⇒ h( x ) ≤ + = 9, ∀x ∈ [ −1,1] 6 3 ច! @ f ( x) = 4 x3 − 3x * ∀x ∈ [ −1,1] Fង x = cos t យ)ង,ន f ( x ) = cos 3t ≤ 1 9)យ max 3ax 2 + 2bx + c = max 12 x 2 − 3 = 9 [ −1,1] [ −1,1] Jញ,ន ច! នBន ថ k : ចប!ផ. ;C8 ផ0ង] # 0 ! ) 8!f # គI 9 2012-11-17បកែ បេ យៈ ែកវ សិរ Page 13
  • 15. បជុំវ សិស ពូែកគណិតវទ ក់ទ១០ ី +. Fង A ', B ', C ' PងW6ច!ន.ចក - 8 BC , CA, AB * OA ' = OB ' = OC ' = R / 2 2π Q Fង i 6<.ចទ| •កF ប # (d ) ( ) Fង i , OA′ = α , x = 3 * A ( i , OB′) = ( i , OA′) + (OA′, OB′) = α + x + k 2π ( i , OC′) = ( i , OA′) + (OA′, OC′) = α − x + k 2π C' O B' N  OA ' 4  OB ' 4  OC '  4  P 1 T= 2   +  +    R   OM   ON   OP     M C   B A' 2 16 = cos 4 α + cos 4 (α + x) + cos 4 (α − x)  4   R 4 = (1 + cos 2α ) 2 + (1 + cos(2α + 2 x)) 2 + (1 + cos(2α − 2 x)) 2  4   R 4 = [3 + 2(cos 2α + cos(2α + 2 x) + cos(2α − 2 x))] + R4 4 + cos 2 2α + cos 2 (2α + 2 x) + cos(2α − 2 x))  4   (1) R 2sin x [ cos 2α + cos(2α + 2 x) + cos(2α − 2 x ) ] = = [sin(2α + x ) − sin(2α − x) + sin(2α + 3 x ) − sin(2α + x) + sin(2α − x) − sin( 2α − 3 x ) ] = sin(2α + 3x) − sin(2α − 3x) = sin(2α + 2π ) − sin(2α − 2π ) = 0 (2) 1 cos 2 2α + cos 2 (2α + 2 x) + cos 2 (2α − 2 x) = [3 + cos 4α + cos(4α + 4 x) + cos(4α − 4 x)] 2 2sin 2 x [ cos 4α + cos(4α + 4 x ) + cos(4α − 4 x )] = [sin(4α + 4π ) − sin(4α − 4π )] = 0 (3) 4  3  18 Fម (1),(2),(3) ⇒ T = 4  3 +  = 4 មQន; ប ប[8 R  2 R =. (8!f # ន g<,ន ជ) 6<Q>a បcង- ម)8ច! 8)យ o<Q>a បcង) '()& 2012-11-17បកែ បេ យៈ ែកវ សិរ Page 14
  • 16. បជុំវ សិស ពូែកគណិតវទ ក់ទ១០ ី វ1 េសរ បឡងអូ ពិចេវ ត ំ ម)ក់ទី១០ េលកទី XII វ ទី៤ . Fង ជ)ងប ក! ព #ក.-ង ABC យ A ', B′, C ′ គ *ប ម.! ប # A ' B ' C ' 6Lន.គមនMAនប ម.! A, B, C យប>ក# 5 ម.!ធ!ប!ផ. ? ប # A ' B ' C ' V/ ង fច ក• )(ន%ងម.! ធ!ប!ផ. ប # ABC ) ព8 ;C8 ក) ន មYព? 2 9 xyz . យប>ក#5 ? xy + yz + zx ≤ + , ក-.ង * x, y , z 6ប ច!នBនពQ មQន 7 7 L< Qជ? ន ផ0ង] #8ក^ 0 _ x + y + z =1 1 1 1 +. គ a, b, c 6ប ច!នBនគ # < Qជ? ន ផ0ង] # 8ក^ 0 _ − = 9)យ d 6 B;ចក a b c មធ! ប!ផ. B ប #ពBកƒ យប>ក#5 abcd នQង d (b − a ) 6ប ? ច!នBន ,កC  xy + yz + zx = 12 =. យប>ក#5 បព| ន} ? ម  នច! 8)យ; មB យគ # ក.-ង !ន.!  xyz − x − y − z = 2 ប Q ច!នBនពQ <ជ? ន យប>ក#5 បព|ន} ? នច! 8)យច! @ x, y , z 6ច!នBនពQ ផ2ងW- '()& ចំេលយ . ពប ច . $ កក-.ង, យ)ង,ន ប % ម.! ប/.នWC: ច បZង - : ម A yប ម.! យ α , β , γ C:ច :ប B' C:ច * យ)ង,ន A ' = 2α , B ' = 2β , C ' = 2γ C' 9)យ A = β + γ , B = γ + α , C = α + β Jញ,ន A ' = B + C − A; B = C + A − B, C = A + B − C B A' C 2012-11-17បកែ បេ យៈ ែកវ សិរ Page 15
  • 17. បជុំវ សិស ពូែកគណិតវទ ក់ទ១០ ី „c:< យ)ង ងR ឃ)ញ5 A' ≥ B ' ⇔ B + C − A ≥ C + A − B ⇔ B ≥ A C:ច * , ប) A ≥ B ≥ C * C ' ≥ B ' ≥ A ' *! A ≤ C' ⇔ A ≤ A + B − C ⇔ B ≥ C : ពQ >a មYព ក) ន ព8 ABC 6 ម|ង2 7 9x 9 xyz . ប) x ≥ , * 1≤ Jញ,ន xy ≤ 9 7 7 2 2 2 6ង ន G ទP ( y + z) ≤ , * xy + xz < < 9 9 7 2 9 xyz C:ច * ក-.ងក ន យ)ងទទB8,ន xy + yz + zx < + 7 7 7 9x „c:<4ប 5 x< , 9)យC:ច * 1 − >0 9 7 (1 − x) 2 យ)ង,ន yz ≤ 4  9 x  (1 − x) 2 2 ព* យ)ង Wន#; g< យប>ក#5 ?  1−  + x(1 − x) ≤  7  4 7 ⇔ (7 − 9 x)(1 − x)2 + 28(1 − x) ≤ 8 . ⇔ 9 x3 + 3x 2 − 5 x + 1 ≥ 0 ⇔ ( x + 1)(3x − 1)2 ≥ 0 . 1 <Q មYព ន ពQ 6នQចj ( @ Q x មQ នL<ជ? ន), ច! @ មYព ក) ន ព8 x = 3 C:ច ន < Q មYព g<,ន យប>ក#, ច! @ ? >a មYព ក) ន8. F; 1 x= y=z= 3 a b c +. Fង A = , B = , C = , C:ច * A, B, C ន B;ចក Bមធ!ប!ផ. គI 1 d d d 1 1 1 យ)ង,ន − = , * AB = C ( B − A) A B C យ)ង យប>ក#5 ( B − A) ? g<; 6ច! នBន ,កC ប) ផ.0យមក< Qញ, គI g< នច! នBន ប…ម p មB យV/ ង |យគ. " ធ!ប!ផ. ប # p ;ចក ច# ន%ង ( B − A) គI p 2 r +1 ច! @ r 2012-11-17បកែ បេ យៈ ែកវ សិរ Page 16
  • 18. បជុំវ សិស ពូែកគណិតវទ ក់ទ១០ ី 6ច! នBន មBយ „c:< ប) p r +1 ;ចក ច# A , * ƒក•;ចក ច# B = ( B − A) + A Jញ,ន p 2 r + 2 ;ចក ច# AB ; p មQន†ច;ចក ច# C ( @ A, B, C មQន នកF ម), B Jញ,ន p 2 r + 2 ;ចក ច# ( B − A) មQន ម 9 .ផ8! C:ច * ;C8†ច G ច * គI B p r ;ចក ច# A , 9)យ យ)ងក•,ន8ទ}ផ8C: ចWច! @ - B ព * ;C8†ច G ចគI B p 2r ;ចក ច# ន%ង AB , ចគI;ចក B ច# ( B − A) មQ ន ម 9 .ផ8! ក * បˆញ5 e ( B − A) 6ច! នBន ,កC ; ABC ( B − A) = ABAB, * ABC ( B − A) g<; 6ច! នBន ABC ( B − A) ,កC, BចគI = ABC ក•6ច!នBន ,កC;C B−A C:ច * (b − a)d = d 2 ( B − A) នQ ង abcd = d 4 ABC .ទ}; 6ច!នBន ,កC =. 6ក#; ង (2,2, 2) 6ច! 8)យមB យ (ក-.ង !ន.!ប ច! នBនពQ <Qជ? ន) ប) យ)ង$ # ទ.ក5 z C:ច6,នC% ង !A8, * គI យ)ង,ន  11z − 2 x + y = z2 + 1    xy = z + 2 z + 12 2   z2 + 1 C:ច * 8ក^ខ _ $!,ច# នQ ង គប# Wន# C)មD x, y , z 6ប ច! នBនពQ < Qជ? នគI (11z − 2)2 > 4( z 2 + 2 z + 12)( z 2 + 1) (*) . 2 (ƒd*,ន5 x នQង y .ទ}; 6ច! នBនពQ ) 9)យ z > ( C) មD x, y នQង z Q .ទ}; <ជ? 11 ន) ព8 * (*) q យ G6 4 z 4 + 8 z 3 − 69 z 2 + 52 z + 44 ≤ 0 ⇔ ( z − 2)2 (2 z + 11)(2 z + 1) ≤ 0 11 1 ក ន g<,ន ផ0ង] # ព8 − 0 ≤ z ≤ − នQង z = 2 2 2 * នH <Qជ? ន; មBយគI z = 2 ( ចព * B x = 2, y = 2 , 6ង ន G ទP យ ; បព|ន} ម ន ឆq. ច! @ x, y នQង z ) 2012-11-17បកែ បេ យៈ ែកវ សិរ Page 17
  • 19. បជុំវ សិស ពូែកគណិតវទ ក់ទ១០ ី ច! @ ប H 6ច!នBនពQ ផ2ង ទP យ)ងគB ពQនQ ម) 8 ង# ប ! A8 Qn oច *q 11 1 ព − G − 2 2 11 ច! @ z = −1 យ)ង,ន,នH មQ ន ផ2ងW (−1, −1, − - ) , $8 2 12 + 2 21 12 − 2 21 ច! @ z = −2 * ,ន x = − ,y=− , ពBកƒ .ទ}; ផ2ងW- 5 5 '()& វ1 េសរ បឡងអូ ពិចេវ ត ំ ម)ក់ទី១០ េលកទី XII វ ទី៥ 1 1 1 . គ Q 3 ច!នBន<ជ? ន a, b, c ផ0ង] # 0 + + =3 យប>ក#5 ? a b c 4 4 4 3 3 3 a 3 + b3 + c 3 ≥ a 2 + b 2 + c 2  x1 + x2 − x3 x4 < 0  ( x1 + x2 )( x3 + x4 ) − x1 x2 − x3 x4 < 0 . យ បព|ន} ម  ( x1 + x2 ) x3 x4 − ( x3 + x4 ) x1 x2 < 0  x1 > 0, x2 > 0, x3 > 0, x4 > 0  +. ក គប#ប ច! នBនគ # ធម(6 Q;C8 ន 8ខបខ0ង# C)មD ច!នBននមB យk6មធ មនព"ន ប #ប ច!នBន;C8Jញ ចញពច!នBន * Q យ<ធច! ~ #ប 8ខ ប #ច!នBន * =. គ *ច! នBន ជង ប #ព9. នQយ| ;C8 នក!ព:8 4 W A, B, C , D ផ0ង] # - 0 1 1 1 ទ!*ក# ទ!នង = + AB AC AD '()& 2012-11-17បកែ បេ យៈ ែកវ សិរ Page 18
  • 20. បជុំវ សិស ពូែកគណិតវទ ក់ទ១០ ី ចំេលយ . Lន.< នM< Q មYពក: ., ពប! Sប#, យ)ង,ន 1 1 1 3= + + ≥ 3 3 abc ⇔ abc ≥ 1 a b c Fង x = 12 a , y = 12 b , z = 12 c 8!f # q យ G6  x > 0, y > 0, z > 0 ច! @  , យប>ក#5 x 9 + y 9 + z 9 ≥ x8 + y 8 + z 8 ?  xyz ≥ 1 Lន.< នM< Q មYពក: . 9 ច! នBន x 9 + ... + x 9 + 1 ≥ 9 x8 (1) y 9 + ... + y 9 + 1 ≥ 9 y 8 (2) z 9 + ... + z 9 + 1 ≥ 9 z 8 (3) យ)ង ន x 8 + y 8 + z 8 ≥ 3 3 x 8 y 8 z 8 ≥ 3 (4) ប:កLងXន%ងLងXប <Q មYព (1), (2), (3), (4) យ)ង,ន8ទ}ផ8;C8 g< យប>ក# ? x 9 + y 9 + z 9 ≥ x8 + y 8 + z 8  A = − x1 − x2 + x3 + x4 > 0 B = x x − x x − x x − x x − x x + x x > 0  . ពQនQ បព| ន}< Q ម 1 2 1 3 1 4 2 3 2 4 3 4  C = x1 x2 x3 + x1 x3 x4 − x1 x3 x4 − x2 x3 x4 > 0  D = x1 x2 x3 x4 > 0  Fង f ( x) = ( x − x1 )( x − x2 )( x + x3 )( x + x4 ) គ. ព*q យ)ង,ន f ( x) = x 4 + Ax 2 + Bx 2 + Cx + D យប មគ. A, B, C , D > 0 * ម f ( x) = 0 មQន†ច នH x < Qជ? នទ C:ច * x1 , x2 ≤ 0 , ផ0.យពប! Sប# C:ច ន បព|ន};C8 គIមQន នH 2012-11-17បកែ បេ យៈ ែកវ សិរ Page 19
  • 21. បជុំវ សិស ពូែកគណិតវទ ក់ទ១០ ី +. ច!នBន;C8 g< ក នSង abc , ច! @ a, b, c ∈ ℕ 9)យ 1 ≤ a ≤ 9, 0 ≤ b, c ≤ 9 bca + cab Fមប! Sប# abc = ⇔ 189a = 81b + 108c 2 ⇔ 7a = 3b + 4c ⇔ 7(a − b) = 4(c − b) (1) . Jញ,ន 4(c − b) ⋮ 7 (2) ⇒ c − b ⋮ 7 (3) . យ 0 ≤ b, c ≤ 9 * −9 ≤ c − b ≤ 9 (4) ព (3) នQង (4) Jញ,ន c − b = −7, 0, 7 ពQ នQ ប ក ក ទ p c − b = −7 ⇔ b = c + 7 ≤ 9 ⇒ c = 0, 1, 2 ⇒ b = 7,8,8 ⇒ a = 3, 4,5 ⇒ abc = 370, 481, 592 . ក ទ p c−b = 0 ⇔ b = c ⇒ a = b = c ⇒ abc = 111, 222, ...., 999 . ក ទ p c−b = 7 ⇔ c =b+7 ≤9 ⇒ b = 0,1,2 ⇒ c = 7,8, 9 ⇒ a = 4,5, 6 . ⇒ abc = 470, 581, 692 . C:ច ន នJ!ងL # 15 ច! នBន;C8 g< កគI 370, 481, 592, 470, 581, 692, 111, 222, 333, ..., 999 =. 4ប 5ព9. $ កក-.ង ង"ង#ផQj % O ! R α Fង α = AOB ( 00 < α < 1200 ) ង# OH ⊥ AB , Jញ,ន AB = 2 HB = 2 R sin 2 3α C:ចW;C - AC = 2 R sin α , AD = 2 R sin 2 1 1 1 ជ!នB ច:8ប! Sប# = + α sin α sin 3α sin 2 2 2012-11-17បកែ បេ យៈ ែកវ សិរ Page 20
  • 22. បជុំវ សិស ពូែកគណិតវទ ក់ទ១០ ី 3α α 3α  C:ច * sin α sin − sin  sin α + sin =0 2 2 2  1 α 5α  1  α 3α  1  cos − cos  −  cos − cos  − ( cos α − cos 2α ) = 0 2 2 2  2 2 2  2  3α   5α  I  cos + cos 2α  −  cos α + cos =0  2   2  7α α α cos sin sin = 0 4 4 2 ច! @ 8ក^ខ _ 0 < α < 1200 , យ)ង,ន 7α 7α 3000 cos =0 ⇒ = 900 ⇒ α = 4 4 7 C:ច ន ព9. ន ជងច!នBន 7 '()& វ1 េសរ បឡងអូ ពិចេវ ត ំ ម)ក់ទី១០ េលកទី XII វ ទី៦ . a). គ Q 3 ច!នBន<ជ? ន;C8 នផ8ប: ក )( 4 យប>ក# 5ផ8ប:កAនព ច! នBន ? ក• យក-.ង 3 ច!នBន * គI មQន :ច6ផ8គ. Aន 3 ច!នBន * ទ b). គ * S = sin 390 + sin 690 + sin1830 + sin 2130 1+ 3 x . a). យ ម −1 = 0 4x + 2 + x b). Fង x, y PងW6 ˆ - " #ម.!J!ងព ក-.ងព9. នQ យ| D1 នQង D1 យC%ង 5 5x − 7 y = 0 កច! នBន ជង ប # D1 , D2 +. Q យប>ក#5 ច! @ គប# ច!នBនគ #<ជ? ? ន n គ,ន 1 1 1 1 + 3 + 3 + ... + <3 2 3. 2 4. 3 (1 + n).3 n 2012-11-17បកែ បេ យៈ ែកវ សិរ Page 21
  • 23. បជុំវ សិស ពូែកគណិតវទ ក់ទ១០ ី =. គ ABC ម, ង# A យC%ង5 L : ង# H ប # Qn o 8) ង"ង#$ %កក-.ង ប # * ច: គ * cos A '()& ចំេលយ . a). Fមប! Sប# គ 3 ច!នBនពQ a, b, c នQង a + b + c = 4 យមQន ធ") , #8ក^ ទ: G, យ)ង យប>ក#5 ? a + b ≥ abc ព (a + b) 2 ≥ 4ab Jញ,ន (a + b + c) 2 ≥ 4(a + b)c ⇔ 16 ≥ 4(a + b)c ⇔ 16(a + b) ≥ 4(a + b)2 c ≥ 16abc . ⇔ a + b ≥ abc . មYព ក) ន ព8 a = b = 1, c = 2 b). យ)ង ន S = 2sin 540 cos150 + 2sin1980 cos150 . = 2cos150 ( sin 540 + sin1980 ) = 2 cos150 ( sin 540 − sin180 ) = 4cos150 cos360 sin180 4cos150 cos360 sin180 cos180 2cos150 cos360 sin 360 = = cos180 cos180 cos150 sin 720 = 0 = cos150 sin 72 6+ 2 S= 4 . a). 8ក^ខ _ x≥0 ម ;C8 1 + 3 x − 4x − 2 + x = 0 ⇔ 3 x − 2 + x = 4 x − 1 ⇔ 8 x − 2 = (4 x − 1) 3 x + 2 + x    4 x − 1 = 0 ⇔ (4 x − 1) 3 x + 2 + x − 2  = 0 ⇔    3 x + 2 + x = 2 2012-11-17បកែ បេ យៈ ែកវ សិរ Page 22
  • 24. បជុំវ សិស ពូែកគណិតវទ ក់ទ១០ ី 1 * 4x − 1 = 0 ⇔ x= 4 7−3 5 * 3 x + 2 + x = 2 យ)ង យ,នH x= 8 1 7−3 5 ន-Q ‰ ន ម នH ព គI x = ; x = 4 8 b). Fងច! នBន ជង ប #ព9. នQ យ| D1 , D2 PងW- យ n នQ ង k 8ក^ខ _ Q n, k 6ច! នBនគ #<ជ? ន 9)យ 3 ≤ k ≤ n (n − 2)π (k − 2)π យ)ង,ន ˆ" #ម.!នមB យk ប # D1 គI x= 9)យ ប # D2 គI y = n k 5(n − 2)π 7(k − 2)π 5x − 7 y = 0 ⇔ = n k ⇔ 5nk − 10k = 7nk − 14n . 7n ⇔ 5k + nk = 7 n ⇔ k = n+5 35 ⇔ k =7− n+5 Q យ k 6ច! នBនគ #<ជ? ន * 35 ⋮ ( n + 5) C:ច * ( n + 5) g< )(ន%ង 1, 5, 7 I 35 ; យ k ≥ 3 ⇒ (n + 5) = 35 C:ច ន n = 30 9)យ k = 6 +. Q ច! @ គប#ច!នBនគ #<ជ? ន k , យ)ង ន 1 1 3 k +1 − 3 k 1 − = = 3 k 3 k +1 3 k (k + 1) 3 k . 3 k + 1. ( 3 (k + 1) 2 + 3 k (k + 1) + 3 k 2 ) 1 1 1 1 Jញ,ន −3 > = 3 k k + 1 3 k . 3 k + 1. 3 3 (k + 1) 2 ( 3(1 + k ). 3 k ) 1  1 1  C:ច ន < 3 3 − 3  (1 + k ). 3 k  k k +1  2012-11-17បកែ បេ យៈ ែកវ សិរ Page 23
  • 25. បជុំវ សិស ពូែកគណិតវទ ក់ទ១០ ី 1 1 1 1  1 1 1 1  *! + 3 + 3 + ... + < 3 1 − 3 + 3 − 3 + ... − 3 <3 2 3 2 4. 3 (n + 1). n 3  2 2 3 n +1  =. Fង O 6ផjQ ង"ង#$ កក-.ង % , ន ! r 9)យ K 6ច!ន.ចក 8ប # BC A Fង x 6 ˆ" #ម.! BHK ⇒ x = 900 − 2 Fង y 6 ˆ" #ម.! BOK , យ)ង,ន  A A A 2 y = 1800 − B = 1800 −  900 −  = 900 + ⇒ y = 450 +  2 2 4 BK BK BHK ន tan x = 9)យ BOK ន tan y = = 2 tan x 2r r  A  A A ⇔ tan  450 +  = 2 tan  900 −  = 2cot g  4  2 2 A A A A A A 1 + tan 1 − tan 2 cos + sin cos 2 − sin 2 ⇔ 4 = 4 ⇔ 4 4 = 4 4 A A A A A A 1 − tan tan cos − sin sin .cos 4 4 4 4 4 4 2 A A  A A A A ⇔ sin .cos =  cos − sin  = 1 − 2sin .cos 4 4  4 4 4 4 A A 1 A 2 ⇔ sin .cos = ⇔ sin = 4 4 3 2 3 A 8 1 ព * យ)ង,ន cos A = 1 − 2sin 2 =1− = 2 9 9 '()& 2012-11-17បកែ បេ យៈ ែកវ សិរ Page 24
  • 26. បជុំវ សិស ពូែកគណិតវទ ក់ទ១០ ី វ1 េសរ បឡងអូ ពិចេវ ត ំ ម)ក់ទី១០ េលកទី XII វ ទី៧ . យ ម Zង ម 8) !ន.!ច!នBនពQ x 4 + 2006 x3 + 1006009 x 2 + x − 2 x + 2007 + 1004 = 0 . បˆញ5 ប) x1 , x2 6ប e H ប # ម x 2 − 6 x + 1 = 0 * ច! @ គប# n∈ ℕ ច!នBន x1 + x2 6ច!នBនគ # មBយ;ចកមQ ន n n ច#ន%ង 5 +. ពQនQ ប Q ច!នBនពQ <ជ? ន a, b, c ផ0ង] #8ក^ 0 _ 2006ac + ab + bc = 2006 2 2b 2 3 ក !A8ធ! ប!ផ. ប #ក នŠម P = − + a 2 + 1 b 2 + 20062 c2 + 1 =. គ ច . @យ ABCD - ន, : ចគI AB ង"ង#មBយ #Fម B នQ ង C ប/ ន%ង ជង AD ង# E , ង"ង#មBយ #Fម A នQ ង D ប/ ន%ង ជង BC ង# F ង"ង#J!ងព ន #W- ង#ព ច!ន.ច M នQ ង N យប>ក# 5 ? J!ងព EMN នQ ង FMN ន កmAផ0 ) (W- '()& ចំេលយ 2007 . 8ក^ខ _ x≥− (*) ម មម:8ន% ង 2 x 2 ( x 2 + 2.x.1003 + 10032 ) + 1 2 ( 2 x + 2007 − 2 x + 2007 + 1 = 0 ) 1 ( ) 2 ⇔ x 2 ( x + 1003) 2 + 2 x + 2007 − 1 = 0 2 ( ផ0ង] # (*) ) 0  x( x + 1003) = 0  ⇔  ⇔ x = −1003  2 x + 2007 − 1 = 0  C:ច ន H ប # ម គI x = −1003 2012-11-17បកែ បេ យៈ ែកវ សិរ Page 25
  • 27. បជុំវ សិស ពូែកគណិតវទ ក់ទ១០ ី . + C! ប:ង យ)ង យប>ក# ? ច! @ គប# n ∈ ℕ , ច!នBន x1n + x2 6ច!នBនគ # (*) n ! ) ពQ ច! @ n = 0, n = 1, n = 2 យ)ង,ន x10 + x2 = 1 + 1 = 2 . 0 x1 + x1 = 6; x12 + x2 = ( x1 + x2 )2 − 2 x1 x2 = 62 − 2.1 = 34 . 1 2 2 4ប 5 ! ) (*) ពQ ច! @ n = k − 1 ច! @ n = k , យ)ង,ន x1k + x2 = ( x1 + x2 ) ( x1k −1 + x2 −1 ) − x1 x2 ( x1k − 2 + x2 − 2 ) k k k = 6 ( x1k −1 + x2 −1 ) − ( x1k − 2 + x2 − 2 ) = 5 ( x1k −1 + x2 −1 ) + ( x1k −1 + x2 −1 ) − ( x1k − 2 + x2 − 2 ) (**) k k k k k C:ច * , ប) ( x1k −1 + x2 −1 ) នQង ( x1k − 2 + x2 − 2 ) 6ប k k ច! នBនគ # * x1k + x2 6ច! នBនគ # k ព * Fម< Q$ Lន. ន មគ B Q <QទO * x1n + x2 6ច! នBនគ #ច! @ គប# n n + „c:< យ)ង យប>ក# x1n + x2 ;ចកមQន ? n ច# ន%ង 5 Fម<Qធ យផ0.យព ពQ 4ប 5 នប ច!នBនគ # ធម(6 Q n V/ ង x1n + x2 ;ចក n ច#ន%ង 5 Fង n0 6ច! នBនគ #ធម(6 Q : ចប!ផ. ; x1n0 + x2 0 ;ចក n ច# ន%ង 5 Fម (**) * ផ8Cក ( x1n0 −1 + x2 0 −1 ) − ( x1n0 − 2 + x2 0 − 2 ) ក• n n g<;ចក ច# ន%ង 5 ;C ជ!នB k យ n0 − 1 ក-.ង (**) យ)ង,ន ( ) ( ) ( x1n0 −1 + x2 0 −1 = 5 x1n0 − 2 + x2 0 − 2 + x1n0 − 2 + x2 0 − 2 − x1n0 − 3 + x2 0 − 3 n n n n ) ព * Jញ,ន ( n ) (  n ) ( n  x1n0 − 3 + x2 0 − 3 = 5 x1n0 − 2 + x2 0 − 2 −  x1n0 −1 + x2 0 −1 − x1n0 − 2 + x2 0 − 2  n ) ក• g<;ចក ច#ន%ង 5 ;C ក ន ផ0.យព 4ប 5 n0 6ច! នBនគ # ធម(6 Q : ចប! ផ. ;C8 x1n0 + x2 0 ;ចក n ច# ន%ង 5 C:ច * , ប!Sប# x1n + x2 ;ចក n ច#ន%ង 5 គI មQន ន C:ច ន x1n + x2 ;ចកមQ ន n ច# ន%ង 5 ច! @ គប# n 2012-11-17បកែ បេ យៈ ែកវ សិរ Page 26
  • 28. បជុំវ សិស ពូែកគណិតវទ ក់ទ១០ ី ab bc +. Fមប! Sប# យ)ងJញ,ន ac + + = 1, 2006 2006 យ a, b, c > 0 * ន A, B, C ∈ (0, π ) V/ ង A+ B+C =π A B B C C A ab bc 9)យ យ tan tan + tan tan + tan tan = 1 = ac + + 2 2 2 2 2 2 2006 2006 A b B C * ប) Fង a = tan ; = tan ; c = tan គI យ)ង,ន 2 2006 2 2 2 2 P= − A 1 3 A B C tan 2 + 1 + = 2cos 2 − 2sin 2 + 3cos 2 2 B C 2 2 2 tan 2 + 1 tan 2 + 1 2 2 C C C A−B = cos A + cos B + 3 − 3sin 2 = −3sin 2 + 2sin cos +3 2 2 2 2 C C 1 A− B 1 10 ≤ −3sin 2 + 3sin 2 + cos 2 +3≤ +3= 2 2 3 2 3 3 2  C 1 A−B ព*  3 sin 2 − cos  ≥0  3 2  C A− B C 1 A− B ⇔ 2sin cos ≤ 3sin 2 + cos 2 2 2 2 3 2  A− B cos 2 = 1 A = B   > " = " ក) a ន8. F;  ⇔  C 1 3sin C = cos A − B sin 2 = 3    2 2 1 2 Jញ,ន c= ; a= ; b = 1003 2 2 2 2 10 1 2 C:ច ន max P = ព8 c = ; a= ; b = 1003 2 3 2 2 2 =. Fង I = EF ∩ MN ; K = AD ∩ BC ; P = EF ∩ ( ADF ); Q = EF ∩ ( BCE ) ( ADE ) = (O); ( BCE ) = (O ') . យ)ង,ន PK / (O) = KF 2 = KA.KD (1); PK / (O ') = KE 2 = KB.KC (2) 2012-11-17បកែ បេ យៈ ែកវ សិរ Page 27